Växthuseffekten, enligt William Happer

Gästinlägg #28 av Gösta Pettersson



2019–2020 publicerade fysikerna W. Happer (Princeton Univ., USA) och W. A. van Wijngaarden (York Univ., Canada) två arbeten (H&W 2019, H&W 2020) som rönt stor internationell uppmärksamhet. Arbetena och kommentarer föranledda av dem gav bland annat under 2021 upphov till tre KU-inlägg författade av Ann Löfving-Henriksson (William Happer ger Bjørn Lomborg en rejäl läxa), Jonas Rosén (IPCC scenarier och Professor Happer) respektive Ingemar Nordin (William Happer om växthusgasernas fysik). Inläggens titlar klargör att man anser Happer vara den ur klimatologisk aspekt intressantaste av de två författarna.

Happer

Jag ansluter mig för enkelhets skull till den bedömningen (trots utomordentligt stor respekt för van Wijngaarden’s vetenskapliga kompetens och bedrifter; se hans 51-sidiga CV) och kommer att hänvisa till de resultat och åsikter som uttrycks i H&W 2019 resp. 2020 såsom varande Happer’s. Jag har noterat att dessa Happer’s resultat av tongivande KU-debattörer (t.ex. Ingemar Nordin, Lennart Bengtsson, Leif Åsbrink) anförts som nära nog obestridliga fysikaliska belägg för växthuseffektens signifikans. Jag är av helt annan mening och ska i detta inlägg förklara varför.

I klimatdebatten råder det tyvärr begreppsförvirring rörande vad termen ”växthuseffekt” ska anses stå för. Happer säger sig studera växthuseffekten så som han själv definierar den: Växthusgasers effekt på den strålningsmässiga värmetransporten från jordytan via atmosfären till rymden. I den bemärkelsen är det förvisso så att det föreligger en växthuseffekt. Växthusgaser har självfallet en radiativt betingad effekt på transporten av värme från jordyta till rymd eftersom en signifikant del av transporten sker genom strålning förmedlad av växthusgaser.

Men denna radiativa effekt på värmetransporten är begreppsmässigt väsensskild från vad jag själv med anslutning till FN:s klimatpanels (IPCC:s) definition alltid betraktat som och i detta inlägg fortsatt kommer att beteckna som en växthuseffekt (GreenHouse Effect, GHE): Nämligen den förhöjning av jordytans temperatur som en ökning av atmosfärens halt av växthusgaser enligt IPCC och andra GHE-proponenter anses medföra. Och i den bemärkelsen ger Happer’s arbeten ingalunda några obestridliga belägg för existensen av en växthuseffekt av signifikant storlek.

I det grundläggande arbetet H&W 2019 har Happer efter gedigen utredning av växthusgasers spektrala egenskaper och inverkan på den strålningsmässiga värmetransporten i atmosfären försökt skatta i vilken utsträckning växthusgaser påverkar utstrålningen av värmeenergi från Jorden. Han fann att en fördubbling av luftens koldioxidhalt globalt sett kan förväntas minska utstrålningen med 3 W/m2, om det råder radiativ jämvikt för värmetransport genom strålning. Varvid de radiativa jämviktsförhållandena kalkylerades utgående från förutsättningen att temperaturen i troposfären avtar linjärt med ökande höjd över jordytan i enlighet med Maxwell-Loschmidts gravito-termala teori (se mitt KU-inlägg 2022-05-26), med den ”lapse rate” av 6,5 °C/km som på empiriska grunder anses gälla för standardatmosfären.

Happers egen kommentar till sina resultat är att en kalkylerad utstrålningsminskning med 3 W/m2 inte är mycket att orda om, eftersom den svarar mot mindre än 1 % av den totala globala utstrålningen (vid pass 400 W/m2) och lokala utstrålningsnivåer kan uppvisa årstidsvariationer i storleksordningen 90 W/m2. Men skattningen 3 W/m2 måste rimligen ge någon information av intresse, så låt oss reda ut vari den informationen skulle kunna bestå.

Kan den kalkylerade utstrålningsminskningen tas som belägg för att det existerar en signifikant växthuseffekt i IPCC:s bemärkelse? Nej, definitivt inte! I H&W 2019 förutsätter Happer att jordytans temperatur är fix och lika med 15,5 °C, såväl när CO2-halten antas vara 400 ppm som när den antas vara fördubblad till 800 ppm. Happer förutsätter med andra ord att det inte föreligger någon växthuseffekt så som den definieras av IPCC (förhöjning av jordytans temperatur). Då är det logiskt sett helt utan vett och sans att ta hans resultat som belägg för att det föreligger en sådan växthuseffekt. I den mån någon gör det, så torde det röra sig om bristande insikt om att Happer använder termen växthuseffekt i helt annan bemärkelse än IPCC.

Det riktar uppmärksamheten mot de angivna förutsättningarna för kalkylerna i H&W 2019. Happer utgår som ovan sagt från att troposfärens temperaturer ansluter sig till den av Loschmidt beskrivna gravito-termala temperaturgradienten. Vilken empiriskt befunnits föreligga och enligt Maxwells förklaring existerar på grund av uppkomsten av vad han kallade ”en konvektiv termisk jämvikt”. Ett stationärt termiskt tillstånd där den troposfäriska temperaturgradienten bestäms av gravitationseffekter utan påvisade bidrag från radiativa effekter.

Inom meteorologisk och klimatologisk forskning har man sedan länge allmänt accepterat begreppet ”konvektiv jämvikt” i den Maxwell-Loschmidtska bemärkelsen och ställt det i motsats till begreppet ”radiativ jämvikt” som svarar mot tänkta förhållanden där de troposfäriska temperaturerna enkom bestäms av strålningsbalanser. Det kan inte samtidigt råda både ”konvektiv jämvikt” och ”radiativ jämvikt” i hela troposfären. När Happer använder sig av den Loschmidtska temperaturgradienten förutsätter han att det råder ”konvektiv jämvikt” i hela troposfären. Då är det motsägelsefullt att samtidigt anta att det råder ”radiativ jämvikt” för strålningsmässiga skeenden i troposfären, t.ex. de som deltar i värmetransporten från jordytan till rymden. Det är skäl nog att betrakta Happer’s skattning 3 W/m2 för utstrålningsminskningen som kvantitativt osäker och grundad på premisser som inte befästs föreligga.

Men framför allt vet vi att temperaturen av Jorden i sin helhet måste svara mot ett stationärt termiskt tillstånd där det råder balans mellan instrålningen av energi till och utstrålningen av energi från Jorden. Denna balans representerar en ”radiativ jämvikt” av överordnad betydelse, vilken utgör det ”jämviktstillstånd” som alla processer för värmetransport i troposfären obligatoriskt och underordnat måste tendera att uppnå.

Happer har i sina analyser bortsett från kravet att det måste råda balans mellan in- och utstrålning. Tar man hänsyn till det kravet, så kan man konstatera att växthusgaser enligt IPCC inte har någon signifikant effekt på instrålningen av solenergi till Jorden. Då kan de inte heller ha någon signifikant effekt på utstrålningen av energi från Jorden vad det stationära tillståndet beträffar. Med andra ord är Happers skattning att en fördubbling av luftens koldioxidhalt minskar utstrålningen med 3 W/m2 behäftad med ett kalkylmässigt fel av samma storlek som själva skattningen. Det följer av att vi à priori vet att det rätta värdet måste vara 0 W/m2 för en Jord i stationärt termiskt tillstånd.

Vad jag därför själv primärt utläser av Happers grundläggande analyser i H&W 2019 är att han med hittills oöverträffat god precision (0 ± 3 W/m2) konfirmerar vad man à priori torde kunna utgå från, nämligen att en fördubbling av luftens koldioxidhalt omöjligen kan åstadkomma någon stationär minskning av utstrålningen av värme från Jorden. Mitt beröm av skattningens precision baserar sig på att den totala globala värmeutstrålningen (vars storlek Happer försöker skatta förändringarna av) är mer än hundrafalt större (cirka 400 W/m2). För mig skänker den goda precisionen tilltro till att troposfären befinner sig i ”konvektiv jämvikt” med temperaturer som ansluter sig till den av Loschmidt beskrivna gravito-termala temperaturgradienten, den förutsättning som Happer baserar sina spektrala analyser på.

I H&W 2020 försöker Happer förfina sina kalkyler av växthusgasers effekt på den radiativa värmetransporten från jordytan till rymden. Därvidlag bollar han med ett flertal hypotetiska förutsättningar rörande den troposfäriska temperaturgradientens karakteristika. I ett av alternativen beaktar han till exempel Nobelpristagaren Manabe’s förslag (se mitt KU-inlägg 2022-11-15) att huvudparten av troposfären befinner sig ”konvektiv jämvikt”, men att denna i troposfärens översta del ersätts av en ”radiativ jämvikt” som medför att det kan uppkomma en växthuseffekt av signifikant storlek. När Happer som ett alternativ accepterar Manabe’s förslag kommer han föga förvånande fram till samma skattning av den förmodade växthuseffektens storlek som Manabe gjorde.

Så vad jag generellt utläser av H&M 2019 och H&M 2020 är att Happers analyser av växthuseffektens signifikans ger resultat som är höggradigt beroende av vad man förutsätter rörande den troposfäriska temperaturgradientens karakteristika och ursprung. I det avseendet har klimatologer anslutit sig till endera GHE-skolan eller Maxwellskolan (egna oprofessionella men lättförstådda benämningar), vars olika syn av i detta sammanhang kritisk betydelse kan beskrivas som följer:

Maxwellskolan förutsätter att den troposfäriska temperaturgradienten är av gravito-termalt ursprung och uppvisar temperaturer som avtar linjärt med ökande höjd hela vägen från jordytan upp till tropopausen i enlighet med Loschmidts kvantifiering av den gravito-termala effekten. Därav drar Maxwellskolan den med matematisk stringens berättigade slutsatsen att IPCC:s förstahandsbevis för existensen av en förindustriell ”naturlig växthuseffekt” i själva verket utgör ett bevis för sagda växthuseffekts insignifikans. Med följdslutsatsen att det ej heller lär finnas någon under den industriella eran förstärkt växthuseffekt eller beaktansvärd växthuseffekt över huvud taget (se mina KU-inlägg 2022-05-27 och 2023-05-26).

GHE-skolan förutsätter att den troposfäriska temperaturgradienten inte enbart bestäms av den gravito-termala effekten, utan även av radiativa effekter som enligt Manabe’s nobelprisade syn medför att temperaturavtagandet uppvisar avvikelser från linearitet av sådant slag att det kan uppstå en växthuseffekt. En av mindre professionella (än Manabe) GHE-proponenter frekvent framförd intuitiv mekanistisk förklaring till den förmodade växthuseffektens uppkomst är att ökande troposfärisk halt av växthusgaser leder till en ökning av utstrålningsskiktets höjd, vilket vid bibehållen ”lapse rate” för den troposfäriska temperaturgradienten innebär att jordytans temperatur måste öka.

Några kommentatorer till mitt senaste KU-inlägg Grand Canyon och växthuseffekten (mest artikulerat Leif Åsbrink #83, 94, 95, 99) tog fasta på den senare intuitiva förklaringen och underkände Maxwellskolans argumentering med motiveringen att den inte tar hänsyn till att utstrålningsskiktets höjd är beroende av radiativa effekter. Då har man för det första inte förstått att Maxwellskolans slutsats rörande växthuseffektens insignifikans dras på matematisk hållbara grunder som endast kräver kännedom om utstrålningsskiktets balansskapande temperatur (-18 °C) och inte kräver beaktande av utstrålningsskiktets höjd. För det andra har man förbisett att Maxwellskolans grundförutsättning utesluter möjligheten att utstrålningsskiktets höjd bestäms av radiativa effekter. Det senare följer av att sagda höjd för vilken som helst observerad eller postulerad jordytetemperatur återspeglar och enkelt kan beräknas från den föreliggande temperaturgradientens ”lapse rate”, vilken enligt Maxwellskolans förutsättning helt och hållet bestäms av den gravito-termala effekten och till ingen del av radiativa effekter. 

Om jordytans globala temperatur var 15 °C vid den industriella erans start (säg år 1750) och den gravito-termala temperaturgradientens globala ”lapse rate” sätts till standardatmosfärens 5,5 °C/km, så återfanns det globala utstrålningsskiktet med dess temperatur -18 °C enligt Maxwellskolans grundförutsättning och elementär matematik år 1750 på (15+18)/5,5 = 6 km:s höjd över jordytan. Om Jordens nuvarande globala temperatur ökat till säg 18 °C med bibehållet värde på temperaturgradientens ”lapse rate”, så återfinns utstrålningsskiktet numera på (18+18)/5,5 ≈ 6,5 km:s höjd. I båda fallen bestäms höjden av gradientens ”lapse rate”, dvs. helt och hållet av den gravito-termala effekten utan påvisat beroende av radiativa effekter.

En global uppvärmning av jordytan måste förvisso vara associerad med en ökning av utstrålningsskiktets globala höjd. Det är en ofrånkomlig konsekvens av den gravito-termala temperaturgradientens existens och en följd av jordytans temperaturförhöjning. Maxwellskolans grundförutsättning utesluter möjligheten att denna ökning av utstrålningsskiktets höjd återspeglar en radiativ effekt. Den utesluter möjligheten att växthusgaser radiativt åstadkommer en förhöjning av utstrålningsskiktets höjd som orsakar jordytans temperaturförhöjning.

Vilka faktorer som i realiteten orsakar de sedan årmiljarder ständigt pågående förändringarna av Jordens globala yttemperatur är en fundamental klimatologisk fråga som vi fortfarande är fjärran från att kunna trovärdigt och allsidigt besvara. Men vad Maxwellskolans argument ger belägg för är att växthuseffekter inte är någon faktor av signifikant betydelse i detta sammanhang. Det är ett delsvar av utomordentlig vikt och utomordentligt intresse. Såväl vetenskapligt, som med tanke på de senaste tre decenniernas klimatdebatts alarmistiska fokusering på de förment vådliga effekterna av ökande lufthalter av växthusgaser. 

Vetenskapligt klargör Happer’s analyser hur avgörande synen på den troposfäriska temperaturgradientens ursprung och karakteristika är för synen på växthuseffekters eventuella signifikans. Som ovan beskrivits finns det två motstridiga skolor i det avseendet. Står måhända ord emot ord, så att vi inte kan avgöra vilkendera skola som är trovärdigast? Inte enligt min bedömning av kunskapsläget.

Maxwellskolans grundförutsättning att troposfärens temperatur avtar linjärt med ökande höjd från jordytan upp till tropopausen i enlighet med Loschmidts ekvationer är utomordentligt väl befäst och gediget stödd av såväl teoretiska som empiriska belägg. Förutsättningen svarar mot existensen av den gravito-termala temperaturgradient som teoretiskt påvisats representera det termodynamiskt stabila tillståndet för en gravitationellt påverkad gasmassa i ett energiskt slutet system. Och som empiriskt befunnits existera även i det reella fallet med en energiskt öppen troposfär där det sker en värmetransport från jordytan via luften till rymden. Maxwell döpte detta tillstånd till ”konvektiv jämvikt”, alldenstund han insåg att avvikelser från det triggar en konvektion i vertikal riktning som snabbt tenderar att återställa det termodynamiskt stabila tillståndet. Maxwellskolans grundförutsättning står såväl i samklang med vad man observerat som med vad man teoretiskt kan förvänta sig av termodynamiska skäl.

Kravet på balans mellan mottagen och avgiven strålningsenergi medför att Jorden som helhet kan förväntas och empiriskt har befunnits uppvisa ”ett utstrålningsskikt” med den temperatur (-18 °C) som bör uppkomma för en planet på Jordens avstånd från solen. Jordytans temperatur är enligt Maxwellskolans grundförutsättning ”exakt” så mycket högre än -18 °C som den gravito-termalt uppkomna temperaturgradienten föreskriver. Jordytans temperatur är med andra ord precis den som kan förväntas för en planet på Jordens avstånd från solen när man tar gravitationens termala effekt i beaktande. Det finns ingenting som tyder på att jordytans temperatur är högre än så på grund av växthuseffekter.

GHE-skolans grundförutsättning att det troposfäriska temperaturavtagandet uppvisar avvikelser från linearitet är däremot inte ens i nobelpristagaren Manabe’s version stödd på någon övertygande teoretisk argumentation (se mitt KU-inlägg 2022-11-15). Men framför allt saknas det några som helst empiriska belägg för existensen av de av Manabe och likasinnade företrädare av GHE-skolan förutsagda avvikelserna från linearitet i det troposfäriska temperaturavtagandet. Till och med den alarmistiskt inställda amerikanska rymdfartsstyrelsen NASA framställer i sina modeller temperaturavtagandet som linjärt från jordytan upp till tropopausen för såväl torradiabaten som våtadiabater i enlighet med Maxwellskolans grundförutsättning.

Så enligt min bedömning har det än så länge inte framkommit något som tyder på att det föreligger växthuseffekter av signifikant storlek. Empiriskt saknas hållbara belägg för att förhöjda halter av växthusgaser i troposfären leder till förhöjd global yttemperatur (eller för den delen till en ökning av utstrålningsskiktets höjd). Teoretiskt har fysiker förklarat att det fortfarande inte framlagts någon fysikaliskt acceptabel förklaring till hur en växthuseffekt av signifikant storlek skulle kunna uppstå. Utan att i detalj gå in på fysikernas kritik i det avseendet kan jag konstatera att GHE-förespråkande klimatmodellerare indirekt tycks ge dem rätt.

Ingen av de klimatmodeller, vars förutsägelser det politiska organet IPCC åberopar som stöd för sina bedömningar av växthusgasernas inverkan på framtidens globala temperaturer, baserar sig mig veterligt på beräkningar som utgår från någon specificerad mekanistisk förklaring till hur de förmodade växthuseffekterna anses uppstå. Alla av mig granskade IPCC-stödda klimatmodeller grundar sig enbart på trosföreställningen att det måste föreligga en växthuseffekt som medför att ökande halter av växthusgaser leder till en förhöjning av jordytans temperatur. En effekt som man kvantifierar genom subjektivt parametriserade samband (= påhittade ekvationer utan mekanistisk eller empirisk förankring) och subjektiva val av de parametervärden som avgör hur stark effekten är. Man konstruerar modeller som levererar förutsägelser återspeglande modellerarens övertygelse om att det existerar en GHE-effekt av viss gissad storlek. Men man förmår inte modellmässigt beskriva hur effekten fysikaliskt förmodas uppkomma. Modellförutsägelserna återspeglar inget vetande, utan enbart tro.

Jag är övertygad om att det existerar radiativa växthuseffekter i IPCC:s mening. Men jag har också synnerligen stark anledning att betvivla att de globalt kan vara av signifikant storlek. Jag är fullt beredd att acceptera tanken att det kan finns globala växthuseffekter av signifikant storlek så snart jag påträffar hållbara teoretiska belägg för att så skulle kunna vara fallet och empiriska belägg för att så faktiskt även tycks vara fallet. Några sådana belägg har jag ännu inte påträffat.

Vad Happer haft att anföra i sina arbeten ändrar inget i det avseendet, utan styrker min övertygelse om att de av Maxwellskolan framförda argumenten leder till den ofrånkomliga slutsatsen att det inte finns någon global växthuseffekt av signifikant storlek. Vill man ifrågasätta den slutsatsen, så är man bland annat i ljuset av Happer’s arbeten tvungen att lägga fram hållbara belägg som falsifierar Maxwellskolans grundförutsättning att troposfärens globala temperatur på grund av den gravito-termala effekten avtar linjärt från jordytan upp till tropopausen i överensstämmelse med vad som kan förväntas av termodynamiska skäl. Några sådana falsifierande belägg har jag inte påträffat

Kommentarer

Kommentera längst ner på sidan.

  1. Mats Kälvemark

    Tack Gösta, ditt inlägg är ett underbart ljus i alarmistmörkret. En konsekvens av dina slutsatser om insignifikansen av fossil CO2:s växthuseffekt är att man då måste presentera trovärdiga andra förklaringar till den pågående, observerade värmningen på ca 0,14 grC per decennium. En sådan förklaring är att det handlar om ”naturliga” förändringar på samma sätt som under perioden 1910-1940 och många andra tidigare perioder tillbaka till slutet på istiden för 11000 år sedan. Har du något annat förslag?

  2. Lennart Bengtsson

    En planets växthuseffekt är definitionsmässigt lika med skillnaden mellan värmestrålningen från jordytan minus jordens utstrålning till rymden. Totalt uppgår den till ca 160 W/m2
    Självklart tas det hänsyn till atmosfärens lapse rate i en sådan beräkning. Skulle lapse rate ( eller Maxwell- Loschmidts relation) vara noll skull växthuseffekten försvinna som jag nämnt i mn bok) Lapse rate är nu en fundamental relation är en följd av allmänna gaslagen och hydrostatisk lagen ( vertikalkomponenten av Navier-Stokes rörelseekvation)
    Happers och Wynjgaarden resultat överensstämmer nästan fullständigt med Manabes artikel från 1967.
    Det framgår också tydligt att den ökning av CO2 och andra växthusgaser ger en mindre ökning på någon grad vid en fördubbling av CO2. Härtill kommer återkopplingseffekter med vattenånga som också påverkas av ökad globalstrålning på grund av renare luft och därmed mindre molnighet ( och därmed högre temperatur).
    Det totala växthuseffekten kan lätt beräknas från de data som sammanställts i Copernicusprogrammet ( baserat på dagliga globala analyser från ECMWF unde drygt 80 år)

  3. Joachim

    Stort tack till Gösta och alla ni andra kunniga som oförtrutet går igenom och förklarar vad vi egentligen känner till om den oerhört komplexa process som somliga förenklat kallar ”klimatet”. Jag behöver läsa detta både en och två gånger.

    Vet ni om nye chefen på IPCC, Jim Skea, är vår bundsförvant i detta? Han tycks försiktigt men stadigt vilja manövrera bort från alarmism och panik till fromma för forskning och konstruktiv handling.

    https://www.svt.se/nyheter/utrikes/ipcc

    Man frågar sig också varför SVT, efter minst ett decennium av total fokus på just alarmism och panik, plötsligt släpper fram någon som varnar för denna hållning. Vad har hänt?

  4. Paul Håkansson

    OT

    Nu har rockis rockström fått problem men har får väl sätta sig i roddbåten och ro över till Kina och ta dom i örat, fy fy.

    KINA
    INFÖR EXPORTKONTROLLER FÖR GRAFIT (Direkt)
    2023-10-20 08:36
    STOCKHOLM (Nyhetsbyrån Direkt) Kina har infört exportkontroller för grafit, en råvara som används ibland annat batterier för elbilar.

    Det rapporterar Financial Times.

    Enligt handelsdepartementet kommer exportkontroller att krävas för tre olika sorters grafit, och åtgärden införs på grund av ”nationell säkerhet”.

    Kina är en ledande producent av grafit och stod för cirka 65 procent av utbudet under 2022. Marknadsandelen i en del grafitbaserade produkter är nära 100 procent.

    USA är tillsammans med Sydkorea en av det största importörerna av grafitprodukter, och åtgärden ses som ett led i den eskalerande konflikten om tekniktillgång mellan USA och Kina.

  5. Lasse

    Happer visade också att en ny dubbling till 800 PPM är ännu mindre påverkande.
    Det är detta jag tar fasta på!
    Huset blir inte mer rött med fler strykningar.
    #2 Kanske kunde solens inverkan på temperaturen påpekas lite i en debattartikel 😉
    Vi vet ju att det kring ekvatorn finns mer nederbörd när strålningen ökar, vilket inte ger anledning till oro för globala temperaturen, även om vi får se solen mer här uppe.

  6. stig morling

    Kära vänner! nu på morgonen har MP fått ett nytt språkrör! Det enda konkreta beskedet han kunde ge var att vi måste sänka CO2-halten. Han kanske bör sluta utandas!
    Salve
    Stig

  7. Bjarne Bisballe

    Happer præciserede i et nyligt foredrag at klimasensitiviteten for CO2 er 0,71°C. Det stemmer præcist med at en fordobling af CO2 vil hæmme udstrålingen fra planten med 3,9 W/m2, (og dermed forøge udstrålingen fra jordoverfladen til 397,9 W fra 394) som det fremgår af figur 10 i H&W 2019 – den helt centrale figur.
    Kun 117 af de 154 W global opvarmning fra drivhusgasserne har med atmosfæreopvarmning at gøre. 37 har at gøre med opvarmning af kloden/havvandet, så når de 117 W stiger til 120 W ved en fordobling, så stiger de 37W til 37,9W, så i alt 3,9 W.

  8. Ronnie

    Jag tackar för utmärkt artikel.
    Läser samtidigt på SVTs hemsida följande:
    De tre månaderna vi just har upplevt är de varmaste på omkring 120 000 år, säger Samantha Burgess, biträdande chef på Copernicus, till nyhetsbyrån AFP.
    Samma uttalande står sedan på flera ställen i olika varianter.
    Jag har svårt att få ihop det- 120 000 år är en lång tid och på denna och andra sidor kan man utläsa i diagram m.m. att detta inte kan stämma. Men hur kan det då hänga ihop?
    Vad är det jag inte förstår ? Är det verkligen så att Copernicus uppsåtligen ljuger ? Har samtidigt svårt att tro att en EU myndighet inte kan förklara ett påstående som ovan ? Är det någon som kan förklara hur det hela hänger ihop ?

  9. Mats Kälvemark

    #8 Ronnie
    Med rätta är du skeptisk till 120.000 år.
    Därav är nämligen 110.000 åt istid. Vi sågar denna idioti i nivå med fotknölarna i inlägget på KU i länken nedan. Men SvT och TT fattar ju inte sin egen idioti. 120.000 år har ju en enastående bra skrämseleffekt! Det smäller högt för dem.

    https://klimatupplysningen.se/en-vanlig-dag-i-klimatkampen/

  10. tty

    ”Men framför allt vet vi att temperaturen av Jorden i sin helhet måste svara mot ett stationärt termiskt tillstånd där det råder balans mellan instrålningen av energi till och utstrålningen av energi från Jorden. ”

    Det vet vi inte alls. I kortsiktigt perspektiv gäller det troligen aldrig. På lång sikt, typ årtusenden, gäller det däremot troligen i stort sett. Havets enorma värmekapacitet buffrar även rätt kraftiga svängningar.

  11. Björn

    Rent fysikaliskt så är det logiskt att en luftpelares tyngd är linjärt ökande mot jordytan och därmed på olika höjd över densamma, fås olika gravitationell effekt. Dessutom minskar molekylernas rörlighet horisontellt i riktning mot jordplanet. Detta är vad jag förstår den gravito-termala effekten. Till detta kan tilläggas den oklara effekten av GHE. Med tanke på att man påstår att CO2 är homogent fördelat i atmosfären, till skillnad mot H2Ogas, så är koncentrationen av CO2 därför inte högre närmre jordytan. Däremot ökar tyngden i luftpelaren något med ökande ppm av CO2. I termer av gravito-termal effekt, så ökar härav också trycket mot jordytan i luftpelaren. Ja, visst kan man anta en kombinerad effekt av den gravito-termala och GHE. Men jämfört med den uppvärmning som solen står för i olika skeenden i tiden av vatten, är emitteringen av H2Ogas i atmosfären, med sin variation upp mot 50000 ppm, en faktor svår att undgå i debatten. Detta medför också en variabel gravito-termal effekt och även av GHE, där kvantitativt, relationen mellan CO2/H2Ogas är ca 400 ppm/upp mot 50000 ppm. Mycket är så oklart vetenskapligt ännu, att det behövs mycket av empiriska studier för att skapa klarhet. Modeller i datorer fungera inte eftersom tidsfaktorn framåt i dessas program inte existerar. Enda vägen framåt är empiri även om det tar lång tid.

  12. Jan-Åke

    #8
    Tja…..
    Mer än 90 % av de 120000 åren hade vi Weichselistiden.
    …. därtill var Norrland täckt av bok och ekskogar under Holocenene Climatic Optimum för 5000-8000 år sedan.
    Googla ”Atlantisk tid” den artikeln i Wikipedia sticker i ögonen på klimathetsarna.
    Skynda dig innan den attackeras.

  13. tty

    #2

    Iden mån det finns återkopplingseffekter via vattenånga så innebär det dock också att lapse-raten kommer att minska eftersom luftens värmekapacitet ökar, vilket ger en negativ återkoppling.

  14. Adepten

    #3 Joachim

    När jag såg Erika Bjerström intervjua Jim Skea igår på Aktuellt, fick jag en helt annan uppfattning av honom.
    Han skulle aldrig ha fått uppdraget om han trodde att vi bara ska avvakta och se hur klimatet utvecklas; istället förespråkar han kraftfulla åtgärder mot CO2-utsläppen.
    Hans engagemang och tydliga ställningstagande under intervjun underströk hans övertygelse om att agera snabbt och effektivt för att tackla klimatutmaningarna.
    Han är ingen bundsförvant för varken klimatrealister eller klimatförnekare.

  15. tty

    #12

    ” därtill var Norrland täckt av bok och ekskogar under Holocenene Climatic Optimum för 5000-8000 år sedan.”

    Ek, alm, ask och lind men inte bok. Den kom in senare. Alm finns fortfarande kvar som relikt på enstaka platser. Hur kom de dit om det inte var varmare än nu?

    https://www.artportalen.se/Image/3144027

  16. Joachim

    #14 Adepten
    Jag måste erkänna att jag inte såg intervjun, bara artikeln på svt.se. Kanske både IPCC och SVT bara positionerar om sig när alarmismen verkar ha nått vägs ände?
    Men jag kan ändå tycka Skeas sätt att uttrycka sig får Guterres att framstå som tämligen oprofessionell.

  17. Gösta Pettersson

    #1 Mats Kälvemark
    ”Har du något annat förslag?”

    Nej något tredje gives icke. Är det senaste seklets globala uppvärmning inte av antropogent ursprung, så måste den vara av annat (icke-antropogent, ”naturligt”) ursprung på samma sättt som temperaturförändringarna under mångmiljontals år i förindustriell tid varit det.

  18. Gösta Pettersson

    #2 Lennart Bengtsson

    Begreppsförvirringen i klimatdebatten är (som jag påpekar) stor beträffande vad som ska förstås med termen ”växthuseffekt”. Du har en egen definition av växthuseffekten som skillnaden mellan värmestrålningen från jordytan och jordens utstrålning till rymden, dvs i princip skillnaden i temperatur mellan jordytan och ”utstrålningsskiktet”.

    Ingen betvivlar att denna temperaturskillnad existerar och är mätbar. Vad Maxwellskolan påpekar är att temperaturskillnaden helt och hållet kan förklaras som en gravito-termal effekt utan påvisade bidrag från någon radiativt betingad växthuseffekt i IPCC:s bemärkelse. Temperaturskillnaden är en ren gravitationseffekt enligt Maxwell-Loschmidts teorier. Att definitionsmässigt döpa den till en ”växthuseffekt” är helt obefogat och förvirrande.

  19. Gösta Pettersson

    #10 tty
    ”Det vet vi inte alls”

    Det har du rätt i. Jag borde kanske som Brasklapp t. ex. ha skjutit in ordet ”långsiktigt”. Men jag hoppas ändå andemeningen i min argumentering står klar. Tar man inte hänsyn till kravet på balans mellan in- och utstrålning, så kan Happers resultat tolkas (och har av många tolkats) som att en fördubbling av CO2-halten till 800 ppm åstadkommer en växthuseffekt genom att minska utstrålningen från jorden med 3 W/m2 med bibehållen instrålning. Jorden skulle inte nå något stationärt termiskt tillstånd utan kontinuerligt tillföras 3 W/m2 mer energi än vad den förmår göra sig av med.

    Det skulle förvisso ge upphov till en växthuseffekt i IPCC:s bemärkelse, och en kraftig sådan. Efter cirka 10 000 år skulle allt vatten på jorden ha värmts till 100 °C. Efter ytterligare 10 000 år skulle allt vattnet ha kokat bort. Och eftersom luftens CO2-halt har varit långt högre än 800 ppm under mer än 500 miljoner år, så skulle allt detta ha skett för länge sedan.

    Det gjorde det inte, därför att kravet på balans mellan in- och utstrålning är städse närvarande och trendskapande. Vill man med Happers approach studera växthusgasernas effekt på jordytans temperatur, så måste man rimligen betrakta yttemperaturen som en variabel (vars värde beror av balansen mellan in- och utstrålning) i stället för som en parameter med fixt värde.

  20. GP #19,

    ”Jorden skulle inte nå något stationärt termiskt tillstånd utan kontinuerligt tillföras 3 W/m2 mer energi än vad den förmår göra sig av med.” ”Efter cirka 10 000 år skulle allt vatten på jorden ha värmts till 100 °C. ”

    Det här måste vara ett rent tankefel. Hur menar du att detta skulle vara fysikaliskt möjligt? Energin är konstant oberoende av vilken temperatur – förhöjd eller inte av växthusgaser – jordytan tar emot. Eller menar du att Happer inte kan denna grundläggande fysik?

  21. # Gösta Pettersson
    Det Du framför är för mig själv ytterligare ett klargörande att den s.k. växthusgaseffekten enligt IPCC och flera prominenta förespråkare, inkl. William Happer är en slags fysikalisk missuppning;
    • Först den självklara ordningen att uppvärmning föregår ökad koldioxhalt i atmosfären, övertygande beskrivet och verifierat av långa data av flera forskare, bl.a. Ole Humlum. Temperaturstegring nära jordytan har redan inträffat före ökningen av koldioxid, och därför före ökad IR-initierad strålningsomsättning. Sammaledes gäller för vattenånga i atmosfären som en funktion av konvektiv värmedistribution.
    • Professor William Happer misstar sig, när han omvandlar ökad koldioxidhalt till ’framtida uppvärmning i atmosfären’ (eller förhindrad värmeutstrålning/avkylning från jordytan). Han antar, vilket Du förklarar, att den nära ytvärmen inte ökar, utan istället att koldioxidens haltökning medför en fördröjningseffekt och därmed nedjustering av den konvektiva värmedistributionen (min tolkning). Det kan vara korrekt effektberäknat, men Schuster-Schwarzschildekvationen, som används, har ingen given temperaturfaktor. Således återstår frågan: medför 3W vid fördubbling av koldioxidhalten från 400 till 800 ppmv som fördröjningseffekt en faktisk temperaturkonvektiv effekt, eller är det endast ett uttryck för ökad energiomsättning i atmosfären, jämförbart med ökad entropi? I sammanhanget är i den ofta använda ekvationen för översättning av strålningsförändring till effektförändring (’Lambert’s Lag’) antagna konstanten k i ekvationen k×delta log haltökning/halt ett mysterium. IPCC anger k=5,35, NASA anger 4,96 och om vi omvandlar Happers 3W får vi ett ännu lägre värde. Konstanten verkar inte vara någon konstant utan i bästa fall en ’beroendefunktion’. Lambert’s Lag förefaller obekvämt oprecis, om det är den som används.
    • I tillägg till ovan vill jag påminna om att långtidsdata från CERES avseende avläst LW-up vid TOA relativt beräknade värden från den jordnära dito avlästa kinetiska temperaturen vid samma tidpunkt, vid olika molnighetsförhållanden, visar faktiska värden, som inte alls bekräftar någon växthusgaseffekt, såsom Lennart Bengtsson operationellt definierar densamma. I detta sammanhang vill jag referera till min kommentar #29 i artikeln ’Atmosfäreffekt ist.f. växthuseffekt-men hur mäter man den?’ 2022-11-25. Min kommentar från faktiska data visar en i princip nollexistent klimatkänslighet för koldioxid.
    • Tekniskt kan vi påminna om två förhållanden:
    1) Lapse Rate är en från termodynamikens första lag ’enkelt härledd funktion’, som anger värdet på den höjdvisa temperaturförändringen såsom gravitationskonstanten/alla atmosfärgasers sammanlagda relativa specifika värmekapacitet. När mängden vattenånga/vatten i atmosfären konvektionsmässigt ökar (principiellt en funktion av jordytans värme) minskar Lapse Rate, p.g.a. vattenångans/vattens högre specifika värmekapacitet jmf atmosfärens dito utan vatten, och omvänt ökar Lapse Rate vid haltökning av koldioxid p.g.a dess lägre specifika värmekapacitet relativt atmosfärens dito medräknat koldioxid vid 370 ppmv halt. Således medför ökad koldioxidhalt principiellt en ökad avkylning av atmosfären, minskad kinetisk värmebehållningskapacitet. Trivialt kanhända, men viktigt att förstå innebörden av!
    2) Om vi tar i beaktande atmosfärens ’energitröghet’, uttryckt som dels ’kinematisk viskositet’ (funktion i längdenhet), dels ’dynamisk viskositet’ (funktion i längd & tryckenhet) finner vi att inga som helst växthusgasers IR effekter kan värma upp atmosfären, eftersom dels den kinematiska viskositeten då inte skulle kunna anta ’den räta linjens funktion’, som den i princip anger, varvid då den dynamiska viskositens funktion inte skulle kunna återspegla atmosfärens temperaturprofil.

  22. Gösta Pettersson

    #20 Ingemar Nordin

    Inget tankefel, utan en för mig uppenbar fysikalisk konsekvens av den situation Happer beskriver i H&W 2019.

    För 400 ppm CO2: IN(400) = UT(400) vilket förutsätts ge den postulerade balansskapande yttemperaturen (15,5 °C).

    För 800 ppm CO2: IN(800) = IN(400) ty opåverkad av CO2
    Men UT(800) = UT(400) – 3 W/m2 ty minskad med 3 W/m2
    Varför IN(800) = UT(800) + 3 W/m2

    Dvs ej längre balans, utan jorden tar vid 800 ppm CO2 kontinuerligt emot 3 W/m2 mer energi än den avger, med kontinuerlig uppvärmning som följd. Inte någon fysikalisk omöjlighet, utan en ofrånkomlig konsekvens av beräkningsresultatet att 800 ppm CO2 minskar utstrålningen med 3 W/m2,

    Hur Happer ser på situationen vet jag inte. Men helt uppenbart syftar hans grundläggande arbete inte till att karakterisera växthuseffekten så som den definieras av IPCC (förhöjning av jordytans temperatur), eftersom han förutsätter att någon sådan växthuseffekt inte föreligger.

  23. Thomas P

    Några korta frågor till Gösta:
    1. Vad bestämmer höjden på tropopausen?
    2. Ändras den om man tillsätter mer växthusgaser?
    3. Vad har det i så fall för effekt på marktemperaturen?

  24. JonasW

    Ja, Manabe och Happer/Wijngaarden säger ungefär samma sak. Den direkta effekten av en fördubblad koldioxidhalt (jämfört med 400 ppm) är ca 2 W/m2.
    Lite osäker på var Gösta Pettersons siffra 3 W/m2 kommer ifrån.

    Sen återstår två frågor:
    1- ändras något mer – som t.ex atmosfärens vattenhalt, lapse rate …
    2- hur påverkas siffran 2W/m2 (3 W/m2 ??) av moln?

    2W/m2 extra innebär grovt uppskattat ca 1 C mer för att återställa strålningsbalansen. Det finns ingen ackumulerande effekt i detta. Temperaturen stiger tills att utstrålning är ungefär densamma som instrålning.

    #10 tty

    Håller helt med. Det finns absolut ingen som helst fysikaliik lag som säger att det måste vara momentan balans. Systemet Jorden har en tröghet, och instrålningen (inklusive ”återstrålning”) kan vara högre än ”utstrålning utan att det händer speciellt mycket.
    Det vore snarare en väldigt stor överraskning om utstrålning var exakt lika med instrålning.

    Tycker att några försökt peka på denna termiska tröghet. Det är inte rimligt att någon Watt extra per kvadratmeter ska ge märkbara effekter på några tiotal år. Det tar tid att värma upp jorden.

    Den uppvärmning vi ser tror jag beror på att det antingen inte finns någon uppvärmning – d.v.s. vi ser fel (=felaktiga data), eller att det är något annat som påverkar (t.ex. variation i havets cirkulation).

    En sista kommentar till Lennart Bengtsson – en ändrad marktemperatur och en oförändrad utstrålning är inte ett bevis för ökad växthuseffekt. Titta på osäkerheten i data, och även hur SB-lagen varierar med temperaturföerdelningen.

  25. Sten Kaijser

    Hej Jonas och tty,

    1 W/m^2 ger 3×10^7 Joule per år och kvadratmeter, vilket räcker för att värma 7,5 meter hav en grad. Om all energi hamnar i haven räcker det till 10 meter.

    Det tar alltså några hundra år att värma ”hela havet” en grad.

  26. Henrik Radhe

    Mycket intressant läsning även om det som presenteras är avancerad naturvetenskap för en icke-fysiker. Om jag som lekman drar en slutsats av läsningen är det att ”växthuseffekten” som företeelse inte är övertygande bevisad trots att alla tagit den för given, trots alla hänvisningar till historiska upptäckter av Arrhenius, Tyndell m fl. För att inte nämna trots alla IPCC:s antaganden som baseras på denna effekt, och trots att de flesta av oss sedan skolan lärt oss att Jordens blivit beboelig tack vare växthuseffekten. Stoffet som presenteras i GP:s artikel är nog ogenomträngligt för de flesta alarmister och får därför inte något medialt eller politiskt genomslag, oavsett påståendena i texten är rätt eller fel. Den fråga på ett allmänt plan som jag tycker artikeln väcker, är varför forskare inom klimatologins ämnesfält tar hypotesen om en växthuseffekt för given. De resonemang som GP framför borde vara föremål för mängder av seminarier och konferenser. Vetenskapen är ingen consensus-förening. Kanske är den enkla och banala slutsatsen att klimatfrågan inte har med vetenskapen göra, bara ideologi och politik, vilket drabbat forskarna lika mycket som åsiktsfabrikörerna i stort. Vad kan högljudda tyckare med vetenskapliga anspråk som Maths Nilsson, Rockström, Michael Tjernström, Erik Kjellström m fl anföra? En tröst är att naturen och skapelsen inte bryr sig om människans diskussioner. Den som lever får se vart hän det barkar.

  27. Gösta Pettersson

    #23 Jonas W

    Uppgiften 3 W/m2 har jag hämtat från Happers egen kommentar till sina resultat enligt länken i inläggets femte stycke.

  28. Gösta P #22,

    Ursäkta, men du begår uppenbarligen ett tankefel om du tror att Happer beskriver en ackumulerande uppvärmning vid en dubblerad CO2-halt. Det här får du nog tänka igenom en gång till. Eller inbillar du dig verkligen att Happer & Co är fysikaliska noviser eller idioter?

  29. forts.

    Det Happer & Wijngaarden gör är helt enkelt en rättfram beräkning baserad på strålningsfysik (välkänd bland de flesta fysiker och bekräftat i otaliga experiment) hur mycket en fördubbling av CO2-halten skulle påverka avkylningen av den marknära temperaturen på klotet. Det finns inget konstigt alls i denna beräkning och den stämmer bra med otaliga andra skattningar. Det Happer och Wijngaarden tillför är att deras beräkningar är betydligt noggrannare än andras.

    Vissa är inte övertygade om strålningsfysiken eller kvantmekanikens grunder utan tror att dessa är bluff och båg. Det verkar som om Gösta Pettersson sällar sig till denna skara.

  30. tty

    #24

    ”vilket räcker för att värma 7,5 meter hav en grad”

    Ungefär 7,8-7,9 meter faktiskt. Havsvatten har lägre värmekapacitet men högre täthet än sötvatten. Sammantaget går det åt ca 5% mindre energi per volymsenhet.

  31. Det är alltid intressant att fundera över atmosfären, co2 och återstrålningen – men återstrålningen verkar inte vara lika bra på att värma haven som samma mängd ökad solinstrålning – och någonting behöver värma våra hav ordentligt om någon större klimatförändring skall inträffa.

    Under dom senaste veckorna har arktis havsis växt med ca 2 miljoner km2 och det är verkligen ovanligt kallt på större delen av havsisen däruppe – det förvånar med dessa rekordvarma hav…

    Just ikväll kan havet i sydligaste sverige stiga till nivåer av samma magnitud som julstormen 1902 och nyårsstormen 1904/5.
    Denna östliga vind med storm i byarna är ordentligt ovanlig.
    NAO:n fortsätter hålla mestadels negativ/ djupt negativ fas som under större delen av 2023 – det händer grejer med vädret/klimatet.

    Skulle det vara så att vi fortsätter få en negativ NAO, pga måncykeln, kan det påverka placeringen för dom mer permanenta hög och lågtrycksområdena i atlanten – och vi får mera östliga väderlägen.
    Detta kan leda till ändringar i havscirkulationen över tid och påverka ytliga havsströmmar och därmed våran temperatur.
    Den numera sydliga havsisutbredningen vid Grönland/Island är riktigt intressant – motsäger starkt rapporterna/satellitbilderna om varmt havsvatten – det har pågått i flera år och blir snart svårt att blunda för.

    Vi har en ökad solinstrålning som värmer haven men denna kan underordnas förändringar i NAO och därmed havsströmmar och låg/högtryck – djuphaven bär oerhörda mängder kyla som lätt övertrumfar atmosfärsförändringar – oavsett en ökad återstrålning kan havsströmmar lätt ge oss mycket kallare klimat.

    Måncyklerna påverkar tidvatten och strömmar – vilket vi skall vara glada för.
    Djupa havsströmmar stabiliserar klimatet.

    Haven kyls och värms av dag och natt och årstider – men också av cykliska havsströmmar.

    Den som tror att kallare hav ger kallare somrar i europa kan dock ha fel – historien visar att kalla hav kan ge otroliga värmeböljor i europa men också kalla vintrar.

    Min morfar föddes 1904, innan den stora nyårsstormen och mormor föddes 1897 – vi kan mycket väl kunna komma att ha snarlikt väder/klimat som under deras barndom i europa under kommande årtionden – trots en ökad återstrålning – tack vare måncykler och djuphavets relativa orubblighet.

    Sorry för lång text/ orerande.

  32. David

    #2
    ” En planets växthuseffekt är definitionsmässigt lika med skillnaden mellan värmestrålningen från jordytan minus jordens utstrålning till rymden. Totalt uppgår den till ca 160 W/m2”
    Hur stor är egentligen värmestrålningen från jordytan? Den kinesiska fysikern Yong Zhong mfl har en intressant teori om att denna är nära 0, dvs majoriteten av den värme som lämnar jordytan upp till atmosfären och vidare utåt sker genom konvektion och först på hög höjd och tunnare luft övergår värmeutbytet till strålning. Så länge värmeutbytet sker genom luftens rörelse så har inga växthusgaser någon påverkan. Någon som har någon reflektion om detta? Tycker det finns en logik i hans resonemang.

  33. Lars-Eric Bjerke

    Gösta Pettersson

    Vi hade uppe denna diskussion för något år sedan d.v.s. om gravitationen i sig orsakar temperaturskillnader i en hög vertikal luftpelare. Johan M förenklade problemet till att modellera luftmolekyler i en vertikal mycket hög helt adiabatisk cylinder och fick som beräkningsresultat att vid jämvikt efter lång tid blir temperaturen densamma i hela cylindern.
    Detta stämmer ju också med termodynamikens andra huvudsats d.v.s. att systemet antar sin lägsta energinivå, vilket jag påstår att systemet får vid jämn temperatur.
    Gravitationen har förstås en stor indirekt betydelse för temperaturfördelningen i atmosfären genom att påverka konvektion och andra rörelser i lufthavet.

  34. Gösta Pettersson

    #29 Ingemar Nordin

    Happer gör i H&W 2019 inte alls någon beräkning av hur mycket en fördubbling av CO2-halten skulle påverka avkylningen av den marknära luften (och därmed jordytans temperatur). Han förutsätter att jordytans (och därmed den marknära luftens) temperatur är fix och opåverkad av en fördubbling av CO2-halten. Vad han utgående från denna förutsättning beräknar är hur utstrålningen mot rymden påverkas av en fördubbling av CO2-halten, varvid han finner att den bör minska med 3 W/m2. Jag har ingen anledning att invända mot dessa hans beräkningsresultat, vilka grundar sig på vad jag kallar en ”gedigen utredning av växthusgasers spektrala egenskaper”.

    Men vad jag starkt invänder mot är föreställningen att Happers resultat ger belägg för existensen av en växthuseffekt som har sin mekanistiska förklaring i att fördubblad CO2-halt leder till en minskning av utstrålningen mot rymden med 3 W/m2. Den tolkningen har inget stöd i Happers resultat och är befängd eftersom den skulle innebära att jorden kontinuerligt och ackumulerat uppvärms med 3 W/m2 på grund av bristande balans mellan in- och utstrålning.

    Det var det befängda i den tanken som jag belyste i min kommentar #19 och sedan försökte förtydliga i #22.

    #28 Ingemar Nordin
    ”du begår uppenbarligen ett tankefel om du tror att Happer beskriver en ackumulerande uppvärmning vid en dubblerad CO2-halt.”

    Nej, jag tror inte att Happer beskriver en ackumulerande uppvärmning. Han befattar sig i H&W 2019 inte överhuvudtaget med CO2:s inverkan på jordytans eller troposfärens temperaturer. Men det finns de som tror att han gör det (jämför din beskrivning i #29 ”avkylning av den marknära luften”) och tar hans resultat som belägg för existensen av en växthuseffekt i IPCC:s bemärkelse (uppvärmning av jordytan), bland annat med hänvisning till hans skattning 3 W/m2. Det är den föreställningen som jag betraktar som obefogad och orimlig.

  35. Gösta Pettersson

    #33 Lars-Eric Bjerke

    Den gamla 1800-talskonflikten mellan Maxwell och Loschmidt rörande gravitationens effekt på de höjdmässiga temperaturförhållandena i en cylindriskt innesluten gas har, som du påpekar, då och då tagits upp även i sentida debatt.

    Måhända kan det göra skillnad om gasen är innesluten eller ej, men jag tror inte det. De fåtal experiment som utförts tyder på att det även i en innesluten gas uppstår en gravitationellt betingad temperaturgradient.

    Men framför allt är det empiriskt välbefäst hur en icke-innesluten gasmassa som atmosfären termiskt beter sig. Den är inte isoterm med samma temperatur överallt. Den uppvisar för troposfärens vidkommande temperaturer som avtar linjärt med ökande höjd över jordytan i enlighet med Loschmidts kvantifiering av den gravito-termala effekten. Med en lapse rate som är proportionell mot gravitationskonstanten för såväl torradiabaten som våtadiabaten. Det råder ingen tvekan om att gravitationen har en helt avgörande inverkan på de troposfäriska temperaturförhållandena.

    Även flertalet GHE-proponenter tycks acceptera Maxwellskolans grundförutsättning att lufttemperaturerna avtar linjärt från jordytan upp till tropopausen på grund av den gravito-termala effekten. Vad de haft svårare att anamma är de av Maxwellskolan påvisade konsekvenserna av denna grundförutsättning, dvs att jordytans förhöjda temperatur relativt utstrålningsskiktet balansskapande -18 °C helt och hållet kan förklaras som en gravitationseffekt och till ingen del återspeglar någon radiativt betingad växthuseffekt.

    Men det är en med matematisk stringens ofrånkomlig konsekvens av Maxwellskolans grundförutsättning. Vilket föranlett mig att i mitt inlägg betona vad som vetenskapligt sett är den avgörande skillnaden mellan Maxwellskolans och GHE-proponenternas syn. Är Maxwellskolans grundförutsättning rörande den troposfäriska temperaturgradienten acceptabel eller inte? Och där jag i ljuset av min bedömning av det nuvarande kunskapsläget finner att det är den. Det finns ingen anledning att tro att växthuseffekter ger något signifikant bidrag till jordytans temperatur.

  36. Gösta: En av mindre professionella (än Manabe) GHE-proponenter frekvent framförd intuitiv mekanistisk förklaring till den förmodade växthuseffektens uppkomst är att ökande troposfärisk halt av växthusgaser leder till en ökning av utstrålningsskiktets höjd, vilket vid bibehållen ”lapse rate” för den troposfäriska temperaturgradienten innebär att jordytans temperatur måste öka.

    Din kritik av detta argument synes mig helt absurd. Om vi tänker oss en planet med en atmosfär helt utan växthusgaser, partiklar eller moln så skulle nödvändigtvis all strålning komma från planetytan.

    Skulle mycket små mängder av t.ex. koldioxid tillföras så skulle en del strålning komma från atmosfären. Det är fysikaliskt omöjligt att strålningen från atmosfären skulle vara oberoende av koncentrationen av koldioxid. Hur ser du på förhållandena på en sådan tänkt planet? Yttemperaturens beroende av koldioxidkoncentrationen – och därmed utstrålningsskiktets höjd?

  37. Gösta: H&W 2019 förutsätter Happer att jordytans temperatur är fix och lika med 15,5 °C, såväl när CO2-halten antas vara 400 ppm som när den antas vara fördubblad till 800 ppm. Happer förutsätter med andra ord att det inte föreligger någon växthuseffekt så som den definieras av IPCC (förhöjning av jordytans temperatur). Då är det logiskt sett helt utan vett och sans att ta hans resultat som belägg för att det föreligger en sådan växthuseffekt. I den mån någon gör det, så torde det röra sig om bristande insikt om att Happer använder termen växthuseffekt i helt annan bemärkelse än IPCC.

    Happer gör sin studie i två steg. Först tänker han sig en momentan (genom trolldom) förändrad CO2-halt från 400 till 800 ppm. Lagom många luftmolekyler omvandlas momentant till koldioxid så att atmosfärens massa förblir oförändrad. I detta första steg är alla temperaturer oförändrade. Alltså ”fördubblad CO2-halt, allt annat oförändrat” I detta steg (tankeexperiment om du så vill) blir utstrålningen till världsrymden momentant förändrad, den sjunker med cirka 3 W/m2. Jag hoppas du kan hålla med om det.

    Enligt W&H 2019: ”The forcings due to instantaneous changes of greenhouse gas concentrations can be calculated quite accurately. Temperature changes induced by the forcings are less clearly defined because various feedbacks change the temperature profile of the atmosphere. After doubling CO2 concentrations, a new, steady state will eventually be established by these feedback processes.”

    Det andra steget är snårigare. Det tänkta tillståndet ”allt annat oförändrat” kan omöjligen vara stabilt. Förhållandena måste ganska snabbt förändras så att energibalans upprätthålls. Ein-Eut=lagrad energi.

    Lagring av energi blir alltså initialt 3W/m2 vilket leder till uppvärmning. Atmosfär och land värms tämligen snabbt, hav mycket långsamt. Hur stor uppvärmningen blir vet vi inte, W&H kommer teoretiskt till något mellan 1,4 och 2,2 grader, men då har dom inte tagit hänsyn till molnen och ändringar i luftcirkulationen mm.

    #7 Bjarne Bisballe skrev: ”Happer præciserede i et nyligt foredrag at klimasensitiviteten for CO2 er 0,71°C. ” i ett privat mail januari 2021 skrev Happer: ”WE ONLY CONSIDERED A CLOUDLESS ATMOSPEHRE. BUT OUR TEMPERATURE PROFILES ARE FROM OBSERVATION. WE DON’T REGARD THE IPCC AS VERY RELIABLE; THE MAIN THING THAT GIVES US CONFIDENCE IN OUR MODELING IS FIG. 15, WHERE YOU CAN HARDLY TELL THE DIFFERENCE OF SPECTRAL INTENSITIES MODELED AT THE TOP OF THE ATMOSPHERE AND THOSE OBSERVED FROM SATELLITES. WE ARE WORKING ON CLOUDS NOW. IT IS A MUCH MORE COMPLICATED PROBLEM THAN CLEAR SKIES.” Jag utgår från att W&H nu har fått ett resultat av beräkningar som inkluderar moln. Självklart måste dom vara mycket mer osäkra än beräkningen av den radiativa forcingen som givetvis varierar med plats på jorden och årstid. 3W/m2 är dock ett värde som kommer från en rimlig ansats. Man kan använda MODTRAN för att räkna ut samma sak i molnfri atmosfär och då får man:

    Tropical atmosphere 3,359 W/m2
    Midlatitude summer 3,045 W/m2
    Midlatitude winter 2,386 W/m2
    Subarctic summer 2,449 W/m2
    Subarctic winter 1,759 W/m2

    Ett medelvärde som viktas mot respektive områdes yta hamnar rimligt nära 3W/m2.

    Denna text visar att Gösta missuppfattat vad Happer skriver: Med andra ord är Happers skattning att en fördubbling av luftens koldioxidhalt minskar utstrålningen med 3 W/m2 behäftad med ett kalkylmässigt fel av samma storlek som själva skattningen. Det följer av att vi à priori vet att det rätta värdet måste vara 0 W/m2 för en Jord i stationärt termiskt tillstånd. I det första beräkningssteget är jorden initialt i ett stationärt tillstånd, men efter fördubblingen av CO2 är den INTE det. Därefter sker en relaxation mot ett nytt stationärt tillstånd med högre temperatur. Hur mycket högre råder det delade menigar om, men högre blir den helt säkert.

    När W&H skriver ”radiative-convective equilibrium of the atmosphere” är det detta de syftar på For thermal radiation with strongly absorbed frequencies, greenhouse-gas molecules, cloud particulates and the air that surrounds them are very nearly in local thermal equilibrium at the same temperature. But for frequencies where the optical depth between the cloud and outer space is of order 1 or less the radiation is not in equilibrium with the air. Denna text kommer från deras nya skrift om moln 1 Jun 2022: 2n-Stream Radiative Transfer https://arxiv.org/pdf/2205.09713.pdf Grundläggande teori om de olika processerna för spridning, absorption och emission i moln. Svår!

    Den senaste: Radiation Transfer in Cloud Layers October 17, 2023: https://arxiv.org/pdf/2310.10622.pdf är ännu svårare, 69 sidor teori. Innehåller inga numeriska resultat för jordens atmosfär. Man jobbar hårt med teorin – men därifrån till utsagor om klimatkänsligheten är det en mycket lång väg.

    Här finns en beskrivning av växthuseffekten i klartext enligt W&H: . For Earth and other planets of the Sun, the optical depth can greatly exceed unity at the centers of strong absorption bands. Negligible surface radiation reaches outer space for these strongly absorbed frequencies, and most escaping thermal radiation is emitted by greenhouse gases at altitudes well above the surface. The radiation to space by greenhouse gases cools air parcels. The cooling is most pronounced at higher altitudes where the radiation has a good chance to reach outer space without being absorbed by still higher-altitude greenhouse gases. Therefore, greenhouse gases cause high-altitude air parcels to cool. As the parcels cool, contract and become heavier than the surrounding air, they sink and are replaced by rising parcels of warmer air that float up from below. March 3, 2023 Atmosphere and Greenhouse Gas Primer https://arxiv.org/pdf/2303.00808.pdf Där kan man vidare läsa: As shown in Fig. 7 and Fig. 8, the decrease in upwards flux at the top of the atmosphere, Z(zmp), due to doubling the CO2 concentration, 3 W m−2, is about a 1% decrease of Z(zmp). Doubling the other anthropogenic greenhouse gases, N2O or CH4, results in even smaller changes to the upwards flux. These flux changes are called the instantaneous forcings. If there were no change in absorbed solar radiation the instantaneous forcing would cause heat to build up in the atmosphere and Earth below. This is where the reliability of climate modeling ends, since no one knows just how the complicated climate system of Earth’s atmosphere and oceans will respond to the small forcings. As shown in reference [6], if one assumes no change in solar heating, and that convective heat transfer keeps the troposphere approximately adiabatic, the surface temperature will warm by about 1 C from doubling CO2, and the upper stratosphere will cool by about 8 C.

    Det finns en summary i W&H 3 mars 2023: Increasing carbon dioxide will cause a small additional surface warming. It is difficult to calculate exactly how much, but our best estimate is that it is about 1 C for every doubling of CO2 concentration, when all feedbacks are correctly accounted for. Varifrån 0,71°C kommer ifrån vet jag inte, men givet de senast publicerade skrifterna verkar det inte vara något att ta på allvar. Det kan vara ett värde Happer får ur någon specifik modell, men för den verkliga jorden är resultatet fortfarande osäkert och jag kan inte tro att en god fysiker som Happer skulle ange det med två siffrors noggrannhet.

  38. Per Johansson

    Vi har ingen växthuseffekt.!?– Vår atmosfär har inget tak eller väggar och allt där emellan rör sig. Den allmänna vetenskapen säger att först stiger temperaturen, sedan ökar koldioxiden. Det beror ju på att varmare hav gasar ut CO2 medans kallare hav absorberar CO2. Våra världshav är ju den största värmereservoaren som är större än vår atmosfär. Koldioxid driver inte värme– vilket vi kan se i ett växthus. Det är ju känt att växthusodlare sprutar in ca: 1000-1500 ppm CO2 i sina växthus lite beroende på vad de odlar. Nu kommer frågan…!?Varför är temperaturen densamma i ett växthus som innehåller 1000 ppm CO2– än i det växthus som innehåller vår atmosfärs nuvarande ca:420 ppm CO2…

  39. Lennart Bengtsson

    24

    Detta är förstås riktigt. Jag har också beräknat” växthuseffektens” ändring under de senaste 75 åren där det visar sig att den totala växhueffekten har ökat ca 3 x mer än den direkta effekten.Skillnaden kan bero på andra effekter som värmer upp jordsystemet eller återkopplingar via de olika växthusgaserna.
    Så frågan är helt enkelt vilka andra fysikaliskt rimliga effekter som är empiriskt möjliga. En möjlig sådan är att aerosoleffekten underskattats. En minskad sådan effekt ger de facto en uppvärmning.
    Vi vet dock att uppvärmningen i troposfären och avkylning i stratosfären följs åt.
    Att visa exakt hur mycket växthugaserna ensamma bidrar med är inte möjligt med dagens kunskap. Som IPCC och andra visat så talar vi här om en osäkerhet på en faktor av 2-3.

  40. Gösta Pettersson

    #36 Leif Åsbrink
    ”Din kritik av detta argument synes mig helt absurd”

    Jag förmodar du med ”detta argument” menar förmodandet att det uppstår en växthuseffekt hänförbar till att utstrålningsskiktets höjd ökar med ökande halt av växthusgaser.

    Min kritik av detta förmodande stöder sig på de observationer som gjorts rörande den troposfäriska temperaturgradienten. Är temperaturavtagandet linjärt från jordytan till tropopausen på grund av den gravito-termala effekten (Maxwellskolans grundförutsättning), så kan inte utstrålningsskiktets höjd vara beroende av halten växthusgaser. Även Manabes modell framställer utstrålningsskiktets höjd som gravitationellt bestämd och oberoende av halten växthusgaser. Ej heller har jag påträffat några empiriska belägg för att det föreligger ett samband mellan utstrålningsskiktets höjd och halten växthusgaser.

    Och mitt inlägg handlar om jorden som den är och kan observeras. Hur de av dig nämnda tänkta världarna ser ut har jag inte haft anledning att här beakta.

  41. Gösta Pettersson

    #37 Per Åsbrink
    ”Denna text visar att Gösta missuppfattat vad Happer skriver:”

    Nej, jag har inte missuppfattat vad Happer skriver, men jag har en annan bedömning än du av vad han faktiskt visat och vilka slutsatser man kan dra av hans resultat.

    Och framför allt är jag övertygad om att Maxwellskolans slutsats rörande växthuseffektens insignifikans är sund om man utgår från att deras grundförutsättning om den troposfäriska temperatugradientens ursprung och karakteristika är korrrekt.

    Vill du slå ett avgörande slag till stöd för GHE-tänkandet, så är det enklaste och bästa tillvägagångssättet att påvisa att Maxwellskolans grundförutsättning är inkorrekt.

  42. Peter Stilbs

    Jag har alltid förundrats över dessa hårklyverier med enstaka Watt/m2 hit och dit i utstrålning från Jorden, mot bakgrund av den verkliga bilden – som är fullt observerbar, och exempelvis illustrerad i den första bilden i ett inlägg här för 13 år sedan https://klimatupplysningen.se/varfor-spana-nar-det-finns-experimentella-data-del-ii/ . Molnighet eller icke molnighet betyder ju 100 Watt/m2 skillnad.

  43. TorbjörnR

    Gösta P

    Tack för som vanligt ett bra inlägg där jag köper mycket av ditt resonemang men kanske inte allt rörande de 3 watt/m2.

    Jag undrar över hur du ser på att natt- och vintertemperaturer verkar ha ökat senaste 30-40 åren medan dagstemperaturer har varit nära oförändrade.

    Natt- och vintertemperaturer skulle kanske kunna bero på fördröjd avsvalning när co2 ”återstrålar” enligt hypotesen från IPCC.

    Dvs vi har en värmande effekt när solens SW-strålning är låg eller försumbar.

  44. Munin

    Se sidan 22. Där finns funktionen för den logaritmiskt avtagande inverkan som gäller koldioxid. Hur mycket den inverkar illustreras också i diagrammet. Funktionen anges vara använd av IPCC.

    https://co2coalition.org/wp-content/uploads/2022/06/Happer-Lindzen-SEC-6-17-22.pdf

    Finns det inte vetenskaplig konsensus om detta??? Är detta omstritt? Strålningsfysiken och hur koldioxiden påverkar är väl mer än känd kunskap sedan hundra år tillbaka!

    Det viktiga är dock hur lite effekt det blir fråga om ifall koldioxidhalten fortsätter öka från nuvarande ca 420 ppm. Det går inte att påstå att det finns något överhängande klimathot vid dessa små tillkommande temperaturökningar.

    Dessutom kan väl inte användningen av tillgänglig fossil energi driva upp koldioxidhalten särskilt mycket och det är en tidsskala på hundratals år för att nå t.ex. 600 ppm.

  45. Peter 42

    Jag håller helt med. Den effekt som co2 ensamt ger upphov till är marginell. Precis som Happer et al säger. Det finns ingen anledning att förneka existensen av växthusgaser effekt, hur liten den än är. Till detta kommer återkopplingen och naturliga processer i hav, jordskorpan och i atmosfären.

    Hela den intressanta frågan är om detta kan åstadkomma en katastrofal uppvärmning på grund av just co2, eller om vi bör titta på historiskt klimat. Inklusive istider? Med Happers konklusion så är väl svaret ganska uppenbart. Vi kan glömma co2-effekten och istället titta närmare på solaktivitet havsströmmar, vulkanism och molnbildning.

  46. Gösta Pettersson

    #43 TorbjörnR
    ”Jag undrar över hur du ser på”

    Jag har ingen syn värd att redovisa beträffande det problemet. I mina klimatologiska resonemang är vanligen alla variabler medelvärdesutjämnade i rum och tid så att dygns- och årstidsvariationer filtrerats bort . Vilket jag ibland glömmer att påpeka.

  47. TorbjörnR

    #46 Gösta P

    Helt ok. Fråga är gratis 😊

    Hela klimatfrågan är ju medelvärdesbildning så det innebär att man ibland glömmer andra signifikanta effekter.

    Tex så är ju 3 Watt/m2 i medel ingenting jämfört med vad moln åstadkommer tex enligt Peter S i #42.

    Lätt att drunkna i detaljer och glömma systemet.

    Jag kan ju tycka att Ein – Albedo = Eut är tillräckligt för att förstå att en ev ändring av Eut antagligen beror på ändrat Albedo i första hand.
    Då bortser man givetvis från andra parametrar kring solens påverkan som vi inte förstår tex magnetism och kosmisk strålning. Men de påverkar väl Albedo då de inte i går i Ein ??

    Om Ein, albedo eller Eut inte ändras så är det väl rimligt att tro att en omfördelning av energi mellan jorden och atmosfären är orsaken till tex global uppvärmning eller avkylning.

  48. Björn

    Man kan ju i samband med förändringar i lufttrycket undra hur detta påverkar det gravito-termala utfallet? Ett högtryck som är stationärt en tid, hur påverkar det temperaturen, om det finns ett samband med gravitationens tryck på luftpelaren?

  49. Gösta Pettersson

    #44 Munin
    ”Finns det inte vetenskaplig konsensus om detta??? Är detta omstritt? Strålningsfysiken och hur koldioxiden påverkar är väl mer än känd kunskap sedan hundra år tillbaka!”

    Strålningsfysiken är välkänd sedan länge och växthusgasernas spektrala egenskaper är utomordentligt väl karakteriserade av bl. a. Happer. Vi har god kunskap om vilka strålningseffekter som kan förväntas uppträda i en ”belyst” gasmassa under välkontrollerade experimentella betingelser där man kan försäkra sig om att det råder ”radiativ jämvikt”.

    Problemet är att atmosfären inte är något sådant välkontrollerat system. Övergripande vet vi att jordytans temperatur långsiktigt bestäms av kravet på balans mellan instrålningen av solenergi och utstrålningen av värme från jorden. Vi vet också att större delen av utstrålningen sker från troposfären. Det innebär
    PRO PRIMO: Att temperaturförhållandena i troposfären (och därmed jordytans temperatur) blir beroende av mekanismerna för värmetransport från jordytan till troposfärens utstrålningsskikt. Växthusgaser kan i princip påverka den radiativa delen av värmetransporten och ge upphov till en växthuseffekt.
    PRO SECUNDO: Att den gravito-termala effekten kommer att ha stort inflytande på de uppkomna temperaturförhållandena enligt de principer som beskrivits av Maxwell och Loschmidt som förklaring till den existerande troposfäriska temperaturgradienten.
    Det råder delade meningar om i vilken mån växthuseffekten och den gravito-termlala effekten bidrar till jordytans temperatur (dvs. ingen vetenskaplig konsensus).

    Maxwellskolan hävdar utgående från sin grundförutsättning (se #40) att jordytans temperatur är den som kan förväntas för en planet på jordens avstånd från solen utan någon signifikant förhöjning på grund av växthuseffekter.

    GHE-proponenter har hävdat att Happers resultat ger obestridliga belägg för existensen av en signifikant växthuseffekt, uppkommen genom att växthusgaser minskar utstrålningen från jorden. Det är den bedömningen som jag vänt mig mot i mitt inlägg, bland annat med hänvisning till att det skulle störa balansen mellan in- och utstrålning.

    Nu går jag över till att beakta Leif Åsbrinks kommentar #37, så fortsätt gärna läsandet.

  50. Gösta Pettersson

    #37 Leif Åsbrink
    ”Alltså ”fördubblad CO2-halt, allt annat oförändrat” I detta steg (tankeexperiment om du så vill) blir utstrålningen till världsrymden momentant förändrad, den sjunker med cirka 3 W/m2. Jag hoppas du kan hålla med om det.”

    Det kan jag göra för diskussionens skull, men skattningen 3 W/m2 baserar sig på förutsättningen att det råder radiativ jämvikt vilken förefaller mig tvivelaktig.

    ”I det första beräkningssteget är jorden initialt i ett stationärt tillstånd, men efter fördubblingen av CO2 är den INTE det. Därefter sker en relaxation mot ett nytt stationärt tillstånd med högre temperatur. ”

    Håller med om vad du säger fram till orden ”med högre temperatur”. En grundförutsättning i Happers analys är att systemet befinner sig i vad Maxwell kallade ”konvektionell jämvikt”, så att Maxwell-Loschmidts gravito-termala temperaturgradient är giltig. Maxwell kallade ”jämvikten” konvektionell, eftersom han insåg att temperaturgradienten är självuppehållande så tillvida att avvikelser från den på grund av gravitationen triggar en konvektion som tenderar att återställa gradienten.

    Om fördubblingen av CO2 stör det stationära tillståndet (vilket Happer i andra ordalag påpekar att den gör) så triggas det förvisso en relaxation. I ljuset av Maxwells argument och Happers grundförutsättning att det råder konvektionell jämvikt måste denna relaxation rimligen tendera att återställa det ursprungliga störda tillståndet. Det finns ingen anledning att tro att man skulle hamna i ett nytt stationärt tillstånd med förhöjda temperaturer. För i så fall skulle störningen av det ursprungliga stationära tillståndet kvarstå. Och temperaturförhöjningen skulle relaxeras bort av konvektionen.

    Det är en av många anledningar till att jag betraktar Happers förslag till uppkomsten av en växthuseffekt med stor skepsis. Happer själv försöker kringgå problemet genom att hänvisa till ospecificerade feed-back effekter. Du beskriver själv hur många oklarheter som vidlåder försöken att kvantifiera den förmodade växthuseffekten enligt Happers approach. Det är så många obefästa förmodanden och spekulationer att man storknar.

    Vilket kan jämföras med Maxwellskolans spekulationsfria och på empiriska data stödda falsifiering av hypotesen att jordytans temperatur är signifikant förhöjd av växthuseffekter. De drar berättigat sin slutsats på basis av ett klart specificerat grundantagande (den troposfäriska temperaturgradienten är av gravito-termalt ursprung och leder till linjärt avtagande temperaturer från jordytan upp till tropopausen). Vill man ifrågasätta hållbarheten av deras argumentering, så måste man påvisa att deras grundantagande är felaktigt.

  51. Munin

    # 49

    Jordens gravitationskraft och solens instrålning mot jorden är båda två lika ständigt närvarande i att verka på jordens atmosfär.

    Dock finns det en skillnad i atmosfärens massa vid 400 ppm och t.ex. 800 ppm koldioxid. Det har skett en förflyttning av atomer/molekyler mellan jordens yta (fast och flytande del) och atmosfären (gasformig). Är den gravito-termala effekten alltid lika stor oavsett atmosfärens massa och var atmosfärens yttre sfär finns?

    Strålningsfysiken handlar om att beräkna hur ändringen i atmosfären av mängden koldioxid (vid t.ex 400 ppm resp. 800 ppm) påverkar utstrålningen från atmosfären.

    Finns det någon variation i den gravito-termala effekten vid ändringar i atmosfärens massa?

  52. #51
    Den gravito-termala effekten är förankrad i atmosfärens masscentrum. Huruvida en ökning av koldioxidhalten från 400 till 800 ppmv ändrar atmosfärens masscentrums höjd, låter jag vara osagt. En fråga som diskuterats är huruvida en ökning av koldioxidhalten samtidigt innebär en minskning av exempelvis syrehalt eller påverkar atmosfärens vattenhalt, samtidigt taget i beaktande att atmosfären inte är strikt begränsad såsom ett slutet rum, dvs expansion/kontraktion är möjlig. I vilket fall som helst är förflyttningen av masscentrum i höjdled relativt obetydlig m.h.t atmosfärens stora massa (ca 1,548 ×10^15 ton våtmassa) och knappast matematiskt signifikant m.h.t initialt stor osäkerhet om exakt lokalisering..
    De två av vikt i denna principiella fråga är faktorn påverkan av atmosfärens specifika värmekapacitet i förhållande till förändringen av masscentrums höjd, vilken f.n. finns vid ca 5,05 kms höjd, som generellt medelvärdesutjämnat värde. En ökad koldioxidhalt, ceteris paribus, ökar Lapse Rate och minskar därmed atmosfärens kinetiska värmebehållningskapacitet, medförande avkylning av atmosfären (och principiellt ökning den ytnära temperaturen, vid bibehållen höjd för atmosfärens masscentrum). Dess verkan är m.h.t koldioxidens ringa halt endast principiellt fastslagen, medan dess faktiska verkan är obetydlig, om ens statistiskt signifikant mätbar.
    Dess påverkan på den ytnära jordytemperaturen, exempelvis höjning, beror således på om haltökningen samtidigt ändrar höjden för masscentrum tillräckligt nedåt (koldioxidens molvikt är högre än syre och vattens).
    Den gravito-termala effekten måste således vid det tänkta utfallet, 800 ppmv relativt 400 ppmv, beräknas utifrån en ny generell medelvärdesrelativ specifik värmekapacitet samt utifrån en ev. förändrad höjd för atmosfärens masscentrum.
    I alla händelser kan vi bortse från en sådan temperatureffekt ytnära (0,000 +/-), eftersom den med stor sannolikhet faller innanför ramen för en matematiskt signifikant osäkerhetsnivå, och drunknar i den temperaturvariabilitet, som är en följd av minst tre andra variablers, nämligen solens totala elektromagnetiska tidsvariabla effekt, den geotermiska verkan av tidsvarierande seismisk aktivitet, vilken också påverkar havens, som specifik tidsvarierande värmedistribuerande faktor, ytnära påverkan, mm..

  53. Munin

    # 52

    Tack! Jag tolkar det som att den gravito-termala effekten varierar med atmosfärens massa, men att det i fallet med koldioxiden och de ändringar som den kan ge i atmosfärens massa är så små att det inte finns ett tillräckligt underlag för att kunna räkna ut exakt hur det ändrar Lapse Rate.

    Atmosfärens värmekapacitet är kopplad till atmosfärens massa och variationer i denna massa, men att det är för stor osäkerhet om de verkliga talen för att kunna räkna ut det exakt.

  54. JonasW

    #39 Lennart Bengtsson

    Undrar hur du räknar ut växthuseffekten?

    Jag gissar att du gör så här.

    1- Räknar ut markytans utstrålade effekt från SB-lag och de rapporterade marktemperaturerna.
    2 – jämför med Ceres data som inte indikerar någon ökad utstrålning
    3 -drar slutsatsen att differensen ”aborberas” / ”återreflekteras” i atmosfären.

    Rätta mig gärna om jag har fel. Du kanske använder någon annan metod.

    Om jag har rätt i min förmodan så finns det några punkter jag vill peka på:

    – man kan inte använda medeltemperaturen vid en beräkning av hur utstrålning enligt SB-lagen ökar. Om man använder medeltemperaturen så stämmer det inte. Du kan väl beskriva hur du beräknat utstrålningen från markytan.
    – noggrannheten i mätningar av jordens utstrålning till rymden. Vad är den osäkerheten? Jag har sett siffror på +/- 4 W/m2. Om denna siffra är korrekt så är det i princip omöjligt att säga om utstrålningen till rymden ökat eller inte.
    Vad är din bild av precisionen i mätning av till rymden utgående effekt?

    Frågar därför att jag vill veta. Finns kanske bra svar. I så fall väldig intressant och relevant.

  55. Gösta Pettersson

    #53 Munin

    Den gravito-termala effekten varierar förvisso med atmosfärens massa enligt Loschmidts kvantifiering av effekten. Det finns fullt tillräckligt underlag för att kunna beräkna hur mycket förändringar av atmosfärens innehåll av t. ex. CO2 påverkar den gravitationella temperaturgradientens lapse rate enligt Loschmidts ekvationer. Sådana beräkningar visar att ökade CO2-halter inom diskuterade gränser (säg upp till 10 000 ppm) inte har någon beaktansvärd inverkan på lapse ratens storlek så som den kvantifierats av Loschmidt och empiriskt befunnits vara.

    Likaså har vi fullt tillräckligt underlag för att med stor säkerhet kunna beräkna hur mycket ökningar av atmosfärens CO2-halt påverkar atmosfärens värmekapacitet. Sådana beräkningar visar att föråndringar av CO2-halten inom diskuterade gränser inte kan ha någon beaktanvärd effekt på värmekapaciteten.

    Så beträffande de undringar dina kommentarer ger uttryck för finns det ett välbefäst svar. Ändringar av luftens CO2-halt har inte någon beaktansvärd effekt på den tropofäriska temperaturgradienten med dess mot gravitationskonstanten proportionella lapse rate. Gradienten kan helt och hållet betraktas som en gravitationell effekt.

  56. Mats Växjö

    Hej Gösta

    Kanske lite Off Topic?
    En ökad CO2-halt sägs ge en temperaturökning då utstrålningen sker från en högre och kallare altitud.
    Bör inte denna temperaturökning även, iaf i ngn mån, höja temperaturen i det ”utstrålande skiktet” – med större utstrålning och därmed lägre temperatur som följd?

    Mats Växjö

  57. Mats Växjö

    I denna tråd framgår ju vikten av vilka antaganden och ståndpunkter man väljer.
    Tyvärr finns inga antaganden redovisade i klimatdebatten.

  58. Munin

    # 55

    Tack, då är bilden klar för koldioxid och dess massas påverkan på gravito-termala effekten. Hur förhåller det sig med vattenångan och dess betydelse för variationer i gravito-termala effekten? Vattenångan är extra intressant eftersom en del i uppbyggnaden av klimathotet är att den ska öka i atmosfären och ge upphov till allehanda följdverkningar.

  59. Gösta Pettersson

    #56 Mats Växjö

    Om det i troposfären transient uppstår en radiativt betingad temperaturförhöjnng i förhållande till vad den Loscmidtska temperaturgradienten föreskriver, så kommer detta att trigga en konvektion som tenderar att åteställa gradienten. Det är detta som Maxwell beskrev när han för 150 år sedan införde begreppet ”konvektiv jämvikt”. Det uppstår aldrig någon radiativ jämvikt, vilket är den huvudinvändning man kan resa mot Happers försök att konstruera en växthuseffektsmodell på basis av sina spektrala analyser och förutsättningen att det råder radiativ jämvikt.
    Jämför med vad jag lite mera detaljerat säger i #50.

  60. Gösta Pettersson

    #58 Munin

    Effekten av vattenånga (och kondensationen av vattenånga) på den troposfäriska temperaturgradientens lapse rate har nogsamt utretts av meteorologer som presenterat ekvationer för den så kallade våtadiabaten. Se t. ex. följande länk https://www.sciencedirect.com/topics/earth-and-planetary-sciences/adiabatic-lapse-rate
    och googla själv vidare på t. ex. ”adiabatic lapse rate” om du vill ha mera kött på benen.

    Observera att även våtadiabatens lapse rate förblir proportionell mot gravitationskonstanten och inte inkluderar några termer hänförbara till radiativa effekter av växthusgaser.

  61. Johan M

    #33 Lars-Eric Bjerke

    Jag är fortfarande av den åsikt att den gravito-termala effekten inte existerar. Det låter så naturligt vid en första anblick och jag var själv övertygad. Naturligtvis måste det förhållas sig så att eftersom molekyler som rör sig uppåt förlorar kinetisk energi så är det kallare på högre höjder.

    När jag själv gjorde simuleringar så förvånades jag över att man inte kunde se ett spår av någon sådan effekt.

    Det finns en lång tråd på Climate.etc (https://judithcurry.com/2014/12/01/gravito-thermal-discussion-thread/) om den gravito-termala effekten. Jag hittade denna artikel som kanske ger en bra förklaring:

    https://tallbloke.files.wordpress.com/2012/01/coombes-laue.pdf
    Jag hittade denna artikel som kanske

  62. Håkan Bergman

    Munin #58
    Även om effekten skulle finnas tror jag den blir svår att mäta eftersom det bara är kolet i koldioxiden som ökar atmosfärens massa, syret ca. 75 % av massan fanns redan i atmosfären. Däremot så skapar den del av vår utsläppta koldioxid som leder till högre absorption av koldioxid i havet eller till en ökning av biomassan en massförlust i atmosfären som jag förstår det. Men jag kommer att sova lugnt i natt också.

  63. Gösta Pettersson

    #61 Johan M
    ”Jag är fortfarande av den åsikt att den gravito-termala effekten inte existerar.”
    följt av
    ”Naturligtvis måste det förhållas sig så att eftersom molekyler som rör sig uppåt förlorar kinetisk energi så är det kallare på högre höjder.”

    Det är det senare förhållandet som utgör den gravito-termala effekten.

    Betr. Coombes-Laue artikel som du ger en länk till, så anser författarna att de genom härledning av Ekv (7) har visat att det granskade systemet måste vara isotermt med höjd-oberoende temperaturer. Samtidigt som de säger att barometerformeln (höjdberoendet av trycket) är härledbar från Ekv (7). Vilket betyder att även temperaturen måste vara höjdberoende; barometerformeln kan härledas från Loschmidts ekvationer och allmänna gaslagen. Det handlar bara om två likvärda alternativa sätt att beskriva gravitationens effekt på luftens tillståndsvariabler.

  64. Gösta Pettersson

    #62 Håkan Bergman

    Jag tror du har missuppfattat vad Maxwell-Loschmidt sagt och kvantifierat rörande den gravito-termala effekten. Den verkar alltid på troposfären i sin helhet. Den tillför ingen energi (= värmer inte), utan förklarar och beskriver endast hur den i troposfären befintliga energin fördelar sig i form av höjdmässiga variationer av temperaturen (och trycket). Mindre ändringar av luftens CO2-halt har inte någon beaktansvärd effekt på den tropofäriska temperaturgradienten med dess mot gravitationskonstanten proportionella lapse rate.

    Allt i enlighet med vad jag skrev i första inlägget i min serie av artiklar om Maxwellskolans syn på troposfärens temperaturer:
    https://klimatupplysningen.se/den-gravito-termala-effekten/

  65. Johan M

    #63 Gösta Pettersson

    Jag tror att artikeln har rätt när de beskriver det misstag man gör in sin slutsats att temperaturen naturligtvis måste minska med höjden. Det är naturligtvis så att molekyler som rör sig uppåt förlorar i kinetisk energi men när man mäter temperaturen på en höjd så räknar man naturligtvis bara med de molekyler som faktiskt kom upp på den höjden. Det finns gott om molekyler som inte hade tillräckligt med energi för att komma upp och de är därmed inte med i medeltalet. Resultatet blir att medeltemperaturen på de molekyler som faktiskt befinner sig på en viss höjd blir konstant.

    Fenomenet uppträder dock bara när vi har partiklar som kolliderar. Har vi ett system men enbart en molekyl eller där det är så få molekyler att kollisioner inte inträffar så har vi en gravito-termal effekt. Så fort vi har tillräckligt mycket molekyler med frekventa kollisioner så försvinner den effekten och vi får en temperatur oavsett höjd.

    Jag tror att man ser effekten även i ett litet system. Ta en simulering av 10 partiklar som släpps från halva höjden i en behållare (elastiska kollisioner med väggar och golv). Om vi inte har några kollisioner så kommer en partikel aldrig att befinna sig högre upp än halva höjden. Inför vi kollisioner så kommer partiklar snart att befinna sig i hela behållaren och hastigheten på de partiklar som når högst upp är inte speciellt låg.

  66. Gösta Pettersson

    #65 Johan M
    ”Resultatet blir att medeltemperaturen på de molekyler som faktiskt befinner sig på en viss höjd blir konstant. ”

    Jo, men den konstansen återspeglar ju begreppet ”medeltemperatur”. Att medeltemperaturen på viss höjd skulle vara konstant i bemärkelsen oberoende av höjden stöds inte av artikelns Ekv (7).

    Vad 10-partikelsimuleringen beträffar blir utfallet självfallet beroende av vilka initialvärden du tilldelar partiklarnas hastighet. Partiklar med en initial hastighetskomponenet i riktning uppåt har inga svårigheter att komma högre upp än halva höjden. Hastigheten vertikalt på de partiklar som kommer högst upp måste vara noll när gravitationen får dem att vända. Såvitt de inte initialt har så hög hastighet uppåt att de slipper ut ur gravitationsfältet (dvs har en initial uppåthastighet överstigande flykthastigheten).

  67. Johan M

    #66 Gösta Pettersson

    ”Vad 10-partikelsimuleringen beträffar blir utfallet självfallet beroende av vilka initialvärden du tilldelar partiklarnas hastighet.”

    Det räcker med att vi sprider ut dem lite slumpmässigt på ca halva höjden och släpper dem samtidigt i.e. ingen vertikal eller horisontell rörelse (de kan inte alla ligga på en linje för då kommer de aldrig att kollidera) .

    När partiklarna börjar kollidera så kommer snart några partiklar att ha en vertikal hastighet så att de når ovanför den ursprungliga höjden. Har du en stor mängd partiklar så kommer de snart att sprida sig i hela rummet.

    Naturligtvis så kommer det finnas fler partiklar nära golvet och färre högre upp men temperaturen kommer vara den samma på alla nivåer.

    Hade vi levt på en plattjord med ständig sol och utan några växthusgaser så hade atmosfären haft en konstant temperatur oberoende av höjd.

  68. #50 Gösta Pettersson

    L.Å.:”Alltså ’fördubblad CO2-halt, allt annat oförändrat’ I detta steg (tankeexperiment om du så vill) blir utstrålningen till världsrymden momentant förändrad, den sjunker med cirka 3 W/m2. Jag hoppas du kan hålla med om det.”

    G.P.: Det kan jag göra för diskussionens skull, men skattningen 3 W/m2 baserar sig på förutsättningen att det råder radiativ jämvikt vilken förefaller mig tvivelaktig.

    Det stämmer inte. Skattningen 3W/m2 baserar sig på uppmätta vertikala temperaturprofiler och växthusgaskoncentrationer. Det görs genom enkla funktioner som är anpassade till observerade data. När W&H skriver ”radiative-convective equilibrium of the atmosphere” menar dom det stycke som jag markerat med fetstil i min kommentar #37. Att detta gäller är en självklarhet. Undantag t.ex. omedelbart efter en blixturladdning. Innebörden är att molekylerna hela tiden är utsatta för kollissioner med andra molekyler i luften och därmed är exciterade till energinivåer enligt Boltzmannfördelningen. Alltså, på svenska: På våglängder där absorptionen är så hög att praktiskt taget all strålning absorberas i gas med samma temperatur som den som råder på den plats varifrån strålningen utsänts från är övergången populerad enligt Boltzmannfördelningen. Detta har inte med värmetransport att göra. Det är termodynamisk jämvikt mellan det elektromagnetiska fältet och den kinetiska gastemperaturen på en viss frekvens.

    På våglängder där absorptionen är så låg att strålning har betydande sannolikhet att nå en temperatur som avviker väsentligt från temperaturen i den gas strålningen kommer ifrån råder inte längre termodynamisk jämvikt mellan det elektromagnetiska fältet och den kinetiska temperaturen. Inom det infraröda fönstret är t.ex. strålningstemperaturen uppåt mycket lägre än lufttemperaturen vid marken. Detta gäller givetvis också vid millimetervågor. Titta på figur 5 a) och c) här: https://www.researchgate.net/publication/215646155_Operational_issues_for_passive_millimeter-wave_imaging_systems/figures?lo=1 Jämför med figur 4.

    Vid närmare eftertanke hoppas jag du kan hålla med om påståendet utan reservationer. Själva värdet 3W/m2 har givetvis en osäkerhet, hur stor vet jag inte, men storleksordningen är korrekt.

    L.Å.: ”I det första beräkningssteget är jorden initialt i ett stationärt tillstånd, men efter fördubblingen av CO2 är den INTE det. Därefter sker en relaxation mot ett nytt stationärt tillstånd med högre temperatur.”

    G.P.:Håller med om vad du säger fram till orden ’med högre temperatur’. En grundförutsättning i Happers analys är att systemet befinner sig i vad Maxwell kallade ’konvektionell jämvikt’, så att Maxwell-Loschmidts gravito-termala temperaturgradient är giltig. Maxwell kallade ’jämvikten’ konvektionell, eftersom han insåg att temperaturgradienten är självuppehållande så tillvida att avvikelser från den på grund av gravitationen triggar en konvektion som tenderar att återställa gradienten.

    Om fördubblingen av CO2 stör det stationära tillståndet (vilket Happer i andra ordalag påpekar att den gör) så triggas det förvisso en relaxation. I ljuset av Maxwells argument och Happers grundförutsättning att det råder konvektionell jämvikt måste denna relaxation rimligen tendera att återställa det ursprungliga störda tillståndet. Det finns ingen anledning att tro att man skulle hamna i ett nytt stationärt tillstånd med förhöjda temperaturer. För i så fall skulle störningen av det ursprungliga stationära tillståndet kvarstå. Och temperaturförhöjningen skulle relaxeras bort av konvektionen.

    Jag finner din invändning absurd. Jag resonerar då så här:
    Givet ditt resonemang så skulle en halvering av CO2 från 400 till 200 ppm inte leda till någon temperaturförändring. Så inte heller en halvering från 200 till 100 ppm osv. ner till att atmosfären vore helt fri från koldioxid.
    Strålningen från jorden skulle då inte längre ha ett hål runt 667 cm-1 utan från rymden se ut som en svartkroppsstrålare. Mer tydligt blir det om man betraktar nedåtgående strålning. MODTRAN ger för Midlatitude Winter nedåtgående strålning 211 W/m2 med 400 ppm men bara 178 W/m2 utan CO2. Skillnad 33W/m2. Det är vid en marktemperatur av -1 grad. Provar man med -20 grader blir värdena 135.2 respektive 101.5 ! Klara, molnfria vinternätter skulle bli mycket kallare om det inte funnes någon CO2 i atmosfären. Uppenbart skulle marktemperaturen sjunka betydligt när IR-strålningen uppifrån reduceras med 25%. Det skulle dra med sig mindre strålning från vattenånga eftersom luften skulle bli torrare.

    Du skriver: ”Det är så många obefästa förmodanden och spekulationer att man storknar.” Ja, må så vara, men att CO2 har en värmande effekt ”allt annat oförändrat” och att det då skapade tillståndet relaxerar mot ett nytt jämviktsläge tror jag vi är överens om. Detta nya jämviktsläge kan inte vara identiskt med det ursprungliga, det är en annan atmosfär och exemplet med koldioxfri atmosfär visar övertydligt att stora minskningar av CO2 har stora effekter. Att måttliga förändringar då helt skulle sakna effekt på temperaturen är inte rimligt att bara sådär löst förmoda.

    Du skriver: ”Vilket kan jämföras med Maxwellskolans spekulationsfria och på empiriska data stödda falsifiering av hypotesen att jordytans temperatur är signifikant förhöjd av växthuseffekter. De drar berättigat sin slutsats på basis av ett klart specificerat grundantagande (den troposfäriska temperaturgradienten är av gravito-termalt ursprung och leder till linjärt avtagande temperaturer från jordytan upp till tropopausen). Vill man ifrågasätta hållbarheten av deras argumentering, så måste man påvisa att deras grundantagande är felaktigt.
    Det är ingen som ifrågasätter att den troposfäriska temperaturgradienten är av gravito-termalt ursprung och leder till linjärt avtagande temperaturer från jordytan upp till tropopausen. Maxwellskolan gör ett antagande om varifrån strålningen kommer. Det är däri felet ligger.

    Jag finner det mycket trist att du vägrar svara på min kommentar #36 Hur de av dig nämnda tänkta världarna ser ut har jag inte haft anledning att här beakta. Ditt svar skulle belysa frågan om hur Maxwellskolan tänker sig att strålning lämnar jorden. Jag förstår det inte. Strålar det från atmosfärend tyngdpunkt oavsett vilka gaser den består av? (Det har jag sett någonstans på nätet.)

    I inlägget skriver du: ”En global uppvärmning av jordytan måste förvisso vara associerad med en ökning av utstrålningsskiktets globala höjd. Det är en ofrånkomlig konsekvens av den gravito-termala temperaturgradientens existens och en följd av jordytans temperaturförhöjning.” Javisst. Vidare: ”Maxwellskolans grundförutsättning utesluter möjligheten att denna ökning av utstrålningsskiktets höjd återspeglar en radiativ effekt. Den utesluter möjligheten att växthusgaser radiativt åstadkommer en förhöjning av utstrålningsskiktets höjd som orsakar jordytans temperaturförhöjning.” Det är detta som jag finner alldeles galet. Vilken ”grundförutsättning” avser du?

    Kan du mena detta: ”Jordytans temperatur är enligt Maxwellskolans grundförutsättning ’exakt’ så mycket högre än -18 °C som den gravito-termalt uppkomna temperaturgradienten föreskriver.” Ja! Det är en sanning vi alla är överens om. Vidare: ”Jordytans temperatur är med andra ord precis den som kan förväntas för en planet på Jordens avstånd från solen när man tar gravitationens termala effekt i beaktande. Det finns ingenting som tyder på att jordytans temperatur är högre än så på grund av växthuseffekter.”

    Om atmosfären helt skulle sakna koldioxid? Har ingen alls betydelse? Kan du verkligen på fullt allvar hävda det?

    Vad är det enligt Maxwellskolan som bestämmer utstrålningsskiktets höjd? Det förefaller som att du menar att det är marktemperaturen?? Det är inte korrekt. Sambandet säger inte vad som är orsak och vad som är verkan.

    Utstrålningsskiktets höjd kan i princip mätas genom en analys av strålningen från jorden. Man kan mäta strålningstemperaturen på alla våglängder och därmed se vilken temperatur, och därmed höjd, strålningen kommer ifrån.

    Se figur 1 här: https://skepticalscience.com/print.php?r=35 Strålningstemperaturen har sjunkit med 1,5 grader från 1970 till 1996 vid 667 cm-1, koldioxidens starkaste band. Man kan också se hur andra växthusgaser strålar från högre höjd 1996 än 1970. Strålningen som totalt lämnar jorden har inte förändrats nämvärt så summan av reflekterat solljus (albedo) och ökad värmestrålning pga högre temperatur måste vara oförändrad. ” Thus the paper found ’direct experimental evidence for a significant increase in the Earth’s greenhouse effect’. Nu har det gått 27 år till sedan 1970. Det finns säkert mycket mer detaljerade mätningar, men svårt att hitta eftersom ytterligare bekräftelse har ringa vetenskapligt nyhetsvärde.

  69. tty

    #68

    ”Att detta gäller är en självklarhet.”

    Det är det inte alls. Vore det så skulle det inte finnas något väder, och ingen nederbörd.

    Det gäller i princip och i stor skala, men lokalt avviker atmosfären faktiskt ofta kraftigt från stabiliteten.

    https://en.wikipedia.org/wiki/Atmospheric_instability

  70. #69 tty

    Du menar alltså att detta inte stämmer: ”På våglängder där absorptionen är så hög att praktiskt taget all strålning absorberas i gas med samma temperatur som den som råder på den plats varifrån strålningen utsänts från är övergången populerad enligt Boltzmannfördelningen.” Inget sägs om storleken på den luftvolym inom vilken konstant temperatur råder. I en tromb eller annan våldsam ostabilitet blir volymen väldigt liten. Då kanske det inte finns någon enda våglängd där radiativ-konvektiv jämvikt råder.

    Annorlunda formulerat: På våglängder där strålningens räckvidd är så kort att temperaturen (excitationsgraden) är lika på platsen för emission och absorption sker ingen energitransport genom strålning. (Strålningens intensitet är då lika i båda riktningarna.)

    Jag tror att du liksom Gösta blandar ihop ”radiativ-konvektiv jämvikt” i den mening Happer använder begreppet med ”radiativ jämvikt” som råder i stratosfären. Det ena är ett lokalt fenomen som beror av våglängden/frekvensen. Det andra säger att energitransport genom strålning är den dominerande mekanismen för energitransport inom ett visst område. Fundamentalt olika saker. Har man sysslat med radioastronomi är absorption i atmosfären med tillhörande termisk strålning självklara fenomen. Intensitet beror av frekvens precis som i infrarött. Det är den radiativa komponenten i radiativ-konvektiv jämvikt. Att Maxwell-Loschmidts gravito-termala teori gäller är den konvektiva aspekten

  71. Gösta Pettersson

    #67 Johan M
    ”Hade vi levt på en plattjord med ständig sol och utan några växthusgaser så hade atmosfären haft en konstant temperatur oberoende av höjd.”

    Det tror nu inte jag, även om jag är oförmögen att presentera empiriska belägg till stöd för min tro. Men du kan väl rimligen inte vara omedveten om man med statistisk-mekanisk metodik redan för hundra år sen genomförde betydligt mera kvalificerade ”simuleringar” än dina, vilka bekräftade att den gravito-termala effekten existerar samt leder det utfall som Loschmidt kvantifierat och beskrivit: En temperaturgradient, ingen isotermi.

  72. Gösta Pettersson

    #68 Leif Åsbrink

    Jag upptäckte alltför sent att mitt inlägg fortfarande genererar kommentarer. Jag kommer att beakta dina sentillkomna synpunkter, men mitt svar avges i morgon enligt min förvrängda tiduppfattning, dvs senare i dag eller eventuellt hiskeligt tidigt i morgon.

  73. Johan M

    #71 Gösta Pettersson

    ”betydligt mera kvalificerade ”simuleringar” än dina,”

    hmm, nu är det inte människors beteende eller ens ett virus spridning som vi talar om utan högstadiefysik. Det finns knappast någon tveksamhet i vad som skall hända när två identiska kroppar kolliderar. Det enda man kan ha gjort annorlunda för hundra år sedan är att man infört räknefel.

    Det fina med den gravito-termala effekten är att den skall visa sig utan att vi behöver tillföra energi, räkna med utstrålning, olika gaser ….. den skall visa sig i en väldigt enkel simulering.

    Allt som vi kan förvänta oss stämmer: tryckgradient, bevarande av energi, isotermi utan gravitation och …… inte ett spår av någon gavito-termal effekt när vi slår på gravitationen.

    Man borde kunna förklara vad som händer med bara två partiklar. Låt oss säga att vi har två partiklar som studsar upp och ner från olika höjder. Om vi nu tar ett stort urval bilder och sammanfattar temperaturen på de olika höjderna så kommer vi se en temperatur-gradient.

    Vad händer om de två partiklarna krockar?

    Som ett resultat av krockarna så kommer partiklarna börja studsa runt och .. partiklarna kommer titt som tätt att nå upp till högre höjder. Någonting kommer hända med gradienten – vad?

  74. Gösta Pettersson

    #68 Leif Åsbrink

    Fan vad ilsk jag är. Efter att under fem timmar ha skrivit ihop ett utförligt svar till din kommentar googlade jag slutligen efter en referens till stöd för vad jag anförde. Vilket medförde att mitt svar raderades på KU-sidan. Jävla programmerare som inte lärt sig att ge fan i att radera något utan författarens godkännande.

    Men jag är envis. I morgon (dvs senare idag eller hiskeligt tidigt i morgon) ska jag försöka rekonstruera mitt svar.

  75. Gösta Pettersson

    #68 Leif Åsbrink
    ”Vid närmare eftertanke hoppas jag du kan hålla med om påståendet utan reservationer. Själva värdet 3W/m2 har givetvis en osäkerhet, hur stor vet jag inte, men storleksordningen är korrekt.”
    Under förutsättningen att jordytans temperatur är oförändrad beräknar Happer att en fördubbling av CO2-halten minskar utstrålningen från Jorden med 3 W/m2, dvs. leder till uppvärmning av jorden. Beräkningsresultatet strider såväl mot den angivna förutsättningen som mot kravet på balans mellan in- och utstrålning till jorden, varför jag omöjligen kan betrakta det skattade värdet 3 W/m2 som särskilt trovärdigt.

    Och om du inte känner till skattningens precision, kan du inte uttala dig om skattningens storleksordning. Om skattningens felgränser (standardavvikelse) t. ex. ges av 3 ± 10 W/m2, så är den enda statistiskt hållbara slutsatsen att det skattade parametervärdet inte kunnat påvisas vara signifikant skilt från noll.

    ”Jag finner din invändning absurd.”
    Min invändning är inte mitt eget påfund, utan ett försök att hänvisa till hur Maxwell på 1800-talet beskrev uppkomsten och vidmakthållandet av det som han kallade en ”konvektiv termisk jämvikt”. Den gravito-termala effekten leder till uppkomsten av en troposfärisk temperaturgradient enligt Loschmidts karakterisering av effekten. Störningar av detta termodynamiskt stabila stationära tillstånd (dvs uppkomna avvikelser från den Loschmidtska gradienten) triggar på grund av gravitationen en konvektion som tenderar att återställa gradienten. Det finns ingen som helst anledning att tro att den av dig diskuterade störningen skulle leda till en relaxation i annan riktning än mot ett återställande av den Loschmidtska gradienten. Speciellt inte när man som Happer förutsätter att troposfärens temperaturen ges av denna gradient.

    Det finns inget absurt i detta. Begreppet ”konvektiv jämvikt” i Maxwells bemärkelse tycks mig vara allmänt accepterat inom meteorologin och klimatologin. Manabe gjorde en klar distinktion mellan regioner där han ansåg det råda ”konvektiv jämvikt” (inga radiativa temperatureffekter) och regioner där han ansåg det råda ”radiativ jämvikt” så att en växthuseffekt kunde uppstå.

    ”Att måttliga förändringar då helt skulle sakna effekt på temperaturen är inte rimligt att bara sådär löst förmoda.”
    Det är inget löst förmodande av Maxwellskolan, utan en konsekvens av deras antagande att det råder ”konvektiv jämvikt” med en Loschmidtsk temperaturgradient vars lapse rate återspeglar gravitationen utan signifikanta bidrag emanerande från växthusgasers radiativa effekter.

    ”Maxwellskolan gör ett antagande om varifrån strålningen kommer. Det är däri felet ligger.”
    Maxwellskolan gör inga som helst antaganden om varifrån strålningen kommer (annat än att de väl outsagt försätter att värmen på jorden kommer från solen). Deras slutsatser baserar sig enbart på grundantagandet att den troposfäriska temperaturgradienten är av gravito-termalt ursprung och leder till linjärt avtagande temperaturer från jordytan upp till tropopausen i enlighet med Loscmidts ekvationer som inte innehåller några termer emanerande från strålning.

    ”Vilken ”grundförutsättning” avser du?”
    Ovannämnda grundantagande, vilket du säger att ingen betvivlr.

    ”Jag finner det mycket trist att du vägrar svara på min kommentar #36”
    Du efterlyser min syn på temperaturförhållandena i tänkta världar utan respektive med gradvis ökande smärre mängder växthusgaser i atmosfären. Du lär få finna dig i att jag inte har någon sådan syn att redovisa och inte heller tänker försöka bilda mig en sådan syn. Jag ser alltför många komplikationer för att finna det meningsfullt att försöka. Utan växthusgaser innebär utan vattenånga. Så när ska den tänkta jorden förses med oceaner? Med ökande mängd växthusgaser ska utstrålningen skifta från att dominerande ske från jordytan till att dominerande ske från atmosfären. Enligt vilka principer och vilket tidsförlopp? Och vad troposfärens temperaturer beträffar ska detta hela tiden sammanjämkas med den gravito-termala effekten. Allt detta utan tillgång till empiriska data som kan bekräfta eller förkasta den syn man söker komma fram till. Jag är alltför empiriskt lagd för att ha lust att ägna mig åt den typen av spekulationer.

    ”Om atmosfären helt skulle sakna koldioxid? Har ingen alls betydelse? Kan du verkligen på fullt allvar hävda det?”
    Nej, det skulle det aldrig falla mig in att hävda. Om luften skulle berövas sin koldioxid så skulle allt levande dö, vilket vore en händelse av stor betydelse. Men om du menar betydelse för jordytans temperatur och byter ut ”ingen alls” mot ”ingen signifikant”, så det precis den slutsats jag drar på basis av Maxwellskolans argumentering.

    ”Vad är det enligt Maxwellskolan som bestämmer utstrålningsskiktets höjd? Det förefaller som att du menar att det är marktemperaturen?? Det är inte korrekt. Sambandet säger inte vad som är orsak och vad som är verkan. ”
    Om du vet marktemperaturen, så vet du med Maxwellskolans grundantagande också på vilken höjd du återfinner utstrålningsskiktet med dess -18 °C. Orsaken är existensen av den gravito-termala gradienten med dess lapse rate. Vad det är för verkan du har i sinnet förstår jag inte.

  76. #75 Gösta Pettersson

    ”Om du vet marktemperaturen, så vet du med Maxwellskolans grundantagande också på vilken höjd du återfinner utstrålningsskiktet med dess -18 °C. Orsaken är existensen av den gravito-termala gradienten med dess lapse rate.”

    Javisst. Självklart!

    Nu vet vi inte marktemperaturen. Vad vi tror oss veta är temperaturanomalin 2 m över marken. Jag förmodar att med det menas observerad temperaturanomali nedräknad till geoiden med lokalt gällande laps rate. Vi vet också temperaturanomalin i ”nedre troposfären”, TLT, ett medelvärde av temperaturen inom ett stort höjdområde med centrum runt 650 hPa (Wikipedia.) Vi vet också att temperaturanomalin växer något snabbare 2m över marken än på cirka 3,5 km höjd. 0,18 respektive 0,14 °C/decennium. Ur temperaturanomalin kan vi beräkna anomalin för utstrålningsskiktets höjd. Alternativt kan vi beräkna anomalin för utstrålningsskiktets höjd ur de spektra vi kan observera från satelliter.

    Du skriver: ”Vad det är för verkan du har i sinnet förstår jag inte.” Jag menar:

    1: Utstrålningsskiktets höjdförändringar är förorsakade av förändringar i marktemperaturen.
    Marktemperatur=orsak
    Höjd=verkan

    2: Marktemperaturens förändringar är förorsakade av förändringar i utstrålningsskiktets höjd
    Höjd=orsak
    Marktemperatur=verkan

    Du hävdar att alternativ 1 är sanningen medan att alternativ 2 är ”En av mindre professionella (än Manabe) GHE-proponenter frekvent framförd intuitiv mekanistisk förklaring till den förmodade växthuseffektens uppkomst”

    Genom din vägran att diskutera hypotetiska resonemang såsom vad Maxwellskolan säger om en planet med atmosfär utan växthusgaser – eller ett jordklot utan koldioxid smiter du ifrån frågeställningens kärna.

    Du skriver även ”GHE-skolan förutsätter att den troposfäriska temperaturgradienten inte enbart bestäms av den gravito-termala effekten” Jag begriper inte varifrån du har fått det. Har du missförstått vad W&H avser med ”radiative-convective equilibrium of the atmosphere” ? Anser du att det jag skrivit om den saken i #37 och #68 är felaktigt? I såfall varför?

    För övrigt, beträffande osäkerheten i beräkningen av forcingen vid en fördubbling av CO2 presenterade jag en liten tabell över resultaten från MODTRAN som ger en uppfattning om saken i min kommentar #37. Hur värdet 3,3W/m2 i tropikerna skall medelvärdesbidas med 2,7W/m2 för ”Midlatitude” beror ju på ytan av respektive område. Det mycket lägre värdet för ”Subarctic winter” 1,8 W/m2 kan försummas eftersom ytan är så liten och sommarvärdet är betydligt högre. Du skriver: ”om du inte känner till skattningens precision,…” Tabellen i #37 ger en uppfattning om att det kan röra sig om ett par tiondels grader.

    Detta som du skriver: ”Det finns ingen som helst anledning att tro att den av dig diskuterade störningen skulle leda till en relaxation i annan riktning än mot ett återställande av den Loschmidtska gradienten. Speciellt inte när man som Happer förutsätter att troposfärens temperaturen ges av denna gradien Här tycker jag det syns tydligt var du missuppfattar beräkningen i W&H.

    Beräkningarna utgör ett tankeexperiment.

    Steg ett: Med ”trolldom” fördubblas CO2-halten. Allt annat oförändrat.

    Steg två, sekunden efter trolleriet: Värmeflödet från atmosfären ut mot rymden har minskat med 3w/m2 men flödet från marken uppåt är oförändrat. (Temperaturen är oförändrad.)

    Steg tre, tar tid på grund av termisk tröghet: Temperaturen i övre troposfären stiger på grund av minskat energiflöde ut mot rymden. På grund av den Loschmidtska gradienten sprider sig denna temperaturökning nedåt i atmosfären ända ner till jordytan. Temperaturen stiger ungefär lika mycket i hela atmosfären och på jordens yta och strålningen ut mot rymden ökar från mark såväl som från alla nivåer i atmosfären tills jämvikt med inkommande solenergi återskapats.

    I steg tre finns ett problem. Den höjda temperaturen kan ändra luftfuktigheten vilket skulle påverka strålningen från vattenånga. I detta avseende redovisas två extremer i W&H. Konstant absolut luftfuktighet respektive konstant relativ luftfuktighet. Tanken är rimligen att verkligheten borde ligga någonstans mellan dessa extremer.

    Sedan tillkommer ett antal steg som inte redovisas i W&H.

    Steg fyra: Ändrad temperatur och luftfuktighet leder till ändrat cirkulationsmönster. Temperaturen stiger mycket på höga latituder men obetydligt vid ekvatorn. Den mycket lägre temperaturen vid höga latituder gör att en högre utstrålningshöjd där inte dämpar energiflödet lika mycket som det skulle gjort vid ekvatorn. Strålningen är ju proportionell mot temperaturen upphöjd till fyra. Jag förmodar detta fenomen ingår i gängse klimatmodeller, det måste vara en betydande negativ återkoppling. Höjd temperatur ändrar också balansen mellan atmosfär och hav. Det leder till mer CO2 i atmosfären, en positiv återkoppling som rimligen också finns med i klimatmodellerna.

    Steg fem: Den ökade längden på växtsäsongen på våra latituder gör att albedo minskar då träd är gröna längre och snön ligger kortare tid. Positiv återkoppling.

    Steg sex: Ökad koldioxidhalt ökar förgröningen globalt. Det förstärker effekten av steg fem men är inte återkoppling på koldioxidens temperatureffekter utan en direkt effekt av att växter gynnas av mer CO2.

    Steg1 till och med steg 3 ger enligt W&H en klimatkänslighe mellan 1,4 och 2,3 °C-

    När Happer tar hänsyn till övriga steg, jag har nog missat några ovan, blir resultatet enligt #7 Bjarne Bisballe: ”Happer præciserede i et nyligt foredrag at klimasensitiviteten for CO2 er 0,71°C.

    Men hur man än vrider och vänder på saken är koldioxid i atmosfären värmande.

  77. Gösta Pettersson

    #76 Leif Åsbrink

    Nu hände det ta mig fan en gång till. Allt jag de senaste fem timmarna skrivit ihop som svar på din kommentar raderades när jag lämnade KU-bloggen för en slutkoll av vissa uppgifter.

    Jag ska rekonstruera svaret, så hav tålamod.

  78. Adepten

    #76 och #77

    Tack för ert idoga arbete att övertyga varandra för vad som känns rätt🙂 Era kommentarer har lärt mig en hel del.

    Gösta! Skriv först ner dina kommentarer i Word eller Note. Kopiera sedan till KU så slipper du bli utkastad.

    Förövrigt har jag kommit fram till att urtvättningen av CO2 i atmosfären tycks minska av någon anledning🤔
    Vilka huvudfaktorer beror det på?

  79. Gösta Pettersson

    #76 Leif Åsbrink

    ”Javisst. Självklart!”
    Utmärkt, då bör du också inse att temperaturskillnaden mellan jordytan och utstrålningsskiktet enligt Maxwellskolans grundantagande helt och hållet är en gravito-termal effekt och till ingen del inkluderar några radiativa bidrag. Jordytans temperatur är precis så mycket högre än utstrålningsskiktets som den Loschmidska temperaturgradientens lapse rate föreskriver och till ingen del ytterligare förhöjd av radiativa effekter. Du tycks äntligen har förstått Maxwellskolans argument.

    ”Nu vet vi inte marktemperaturen.”
    Då får vi väl vid behov mäta den och beräkna den. Dina synpunkter ändrar inte ett skvatt på slutsatsen att utstrålningsskiktets höjd kan beräknas från den gravito-termala gradientens lapse rate och temperaturen i vilken som helst lägre belägen del av troposfären, dvs att den (med Maxwellskolans grundantagande) helt och hållet bestäms av gravitationen och till ingen del återspeglar några radiativa effekter.

    Betr. dina två alternativ för orsak-verkan är det rätt att min argumentering utgår från Alternativ 1. Alternativ 2 kan (med Maxwellskolans grundantagande) förkastas, eftersom det leder till en slutsats som står i strid med grundantagandet. Växthuseffekter kan inte leda till en ökning av utstrålningskiktets höjd, eftersom höjden enligt slutsatsen i föregående stycke till ingen del återspeglar några radiativa effekter.

    ”Här tycker jag det syns tydligt var du missuppfattar beräkningen i W&H.
    Beräkningarna utgör ett tankeexperiment.”
    Det såg jag ingenting om i H&W 2019, men jag förstår vad du menar och har inget att invända mot din beskrivning i Steg ett, Steg två och första meningen under rubriken Steg 3. Sen är det tvärstopp för min acceptans av din syn.

    ”Steg tre, tar tid på grund av termisk tröghet: Temperaturen i övre troposfären stiger på grund av minskat energiflöde ut mot rymden. På grund av den Loschmidtska gradienten sprider sig denna temperaturökning nedåt i atmosfären ända ner till jordytan.”
    Om temperaturen i övre troposfären transient ökar, så har det uppstått ett värmeöverskott gentemot vad den Loschmidtska gradienten (utgångstillståndet) föreskriver. Att detta värmeöverskott skulle kunna sprida sig nedåt och förhöja temperaturen i lägre luftlager/jordytan är en omöjlighet enligt termodynamikens andra huvudsats; spontan värmetransport kan endast ske från varmare materia till kallare.

    Maxwell gav redan på 1800-talet en sund förklaring till vad som kan förväntas hända om det någonstans i troposfären uppstår en transient temperaturförhöjning i förhållande till vad den Loschmidtska gradienten förskriver. Det kommer att triggas en konvektion som får den ”alltför varma” luften att stiga (och avkylas) till dess den antar den temperatur som föreskrivs av Loschmidts gradient. Gradienten tenderar att återupprätta sig genom konvektion, vilket var grundskälet till att Maxwell kallade det Loschmidtska grundtillståndet för ”konvektiv termisk jämvikt”. ”Alltför” varm luft stiger på grund av gravitationen uppåt. Den faller inte nedåt.

    ”Men hur man än vrider och vänder på saken är koldioxid i atmosfären värmande.”
    Tror du och andra GHE-proponenter, ja. Men Maxwellskolan har lagt fram hållbara argument för att så inte kan vara fallet om deras grundantagande är giltigt. Och grundantagandet säger du själv att ingen ifrågasätter.

  80. #79 Gösta Pettersson

    Detta måste jag tydligen skriva mer pedantiskt: Steg tre, tar tid på grund av termisk tröghet: Temperaturen i övre troposfären stiger på grund av minskat energiflöde ut mot rymden. På grund av den Loschmidtska gradienten sprider sig denna temperaturökning nedåt i atmosfären ända ner till jordytan.”
    Atmosfären är hela tiden i rörelse. Genom konvektion och även latent värme och i någon liten utsträckning genom strålning transporteras energi uppåt. När koldioxiden momentant ökar blir strålningen ut mot rymden momentant svagare med cirka 3W/m2. Dessa 3W kommer från en stor volym luft, en luftpelare med en höjd af flera km. Det tar på grund av termisk tröghet en avsevärd tid innan någon jämvikt uppnåtts. Det är inte något värmeöverskott som sprider sig nedåt. Det är värmetransporten uppåt som påverkas vilket jag avsåg med ”På grund av den Loschmidtska gradienten.” Som du skriver: ”Gradienten tenderar att återupprätta sig genom konvektion,” Alltså, när temperaturen sakta stiger en aning inom ett flera kilometer tjockt skikt i troposfären kommer konvektionen att se till att gradienten hela tiden upprätthålls.

    ”Om temperaturen i övre troposfären transient ökar, så har det uppstått ett värmeöverskott gentemot vad den Loschmidtska gradienten (utgångstillståndet) föreskriver.” Det är en felaktig beskrivning. Spelar i och för sig ingen roll – man kan anta att det skulle ske momentant. Då skulle den inte längre tillräckligt varma luften i lägre skikt stiga långsammare/mindre så att gradienten återupprättas.

    Spontan värmetransport kan endast ske från varmare materia till kallare men den kan gå saktare ifall den kallare materien blir en aning mindre kall.

    Som du skriver: ” ’Alltför’ varm luft stiger på grund av gravitationen uppåt. Den faller inte nedåt.” – men den kan stiga långsammare eller snabbare beroende på omgivande lufttemperatur.

  81. #79 Gösta Pettersson

    Självklart inser jag att temperaturskillnaden mellan jordytan och utstrålningsskiktet enligt Maxwellskolans grundantagande helt och hållet är en gravito-termal effekt och till ingen del inkluderar några radiativa bidrag.

    Svagheten i ditt resonemang är att det inte på något sätt motsäger att både marktemperatur och utstrålningsskitets höjd kan förändras – och de facto ständigt förändras. Just nu bekymrar sig många över de cirka 1,2 grader som marktemperaturen anses ha ökat med. Mycket större temperaturförändringar har skett under årmiljonerna – och då har rimligen utstrålningsskiktets höjd varierat även om ändringar i albedot och omloppsbanan också varit betydelsefulla.

    Maxwellskolans proponenter har inget antagande om vad som bestämmer marktemperaturen och därmed utstrålningsskiktets höjd. Vi vet att dessa saker förändras över tid – men alltid på ett sätt som överensstämmer med Maxwellskolans grundantagande.

    Detta är inte på något sätt ett bevis för att någon viss hypotes om varför jordytans temperatur förändras är fel.

    Somliga hävdar att orsaken till förändringarna av marktemperaturen beror på förändringar i solstrålningen. Såväl intensitet som spektral fördelning och partikelstrålning. Andra hävdar förändrad molnbildning på grund av kosmisk strålning, det hävdas också inverkan av ändrad halt av koldioxid och andra växthusgaser. Jag har sett ett antal ytterligare mer eller mindre fantasifulla förslag.

    Jag kan inte förstå hur du kan hävda att Maxwellskolans grundantagande kan motbevisa någon av hypoteserna. Att det måste finnas en orsak finner jag självklart. Rimligen finns flera oberoende och samverkande orsaker.